56 inc statement NINJA MCQ Flashcards

1
Q

On October 1 of the current year, a U.S. company sold merchandise on account to a British company for 2,000 pounds (exchange rate, 1 pound = $1.43). At the company’s December 31 fiscal year end, the exchange rate was 1 pound = $1.45. The exchange rate was 1 pound = $1.50 on collection in January of the subsequent year.

What amount would the company recognize as a gain(loss) from foreign currency transaction when the receivable is collected?

A $0

C $140
D ($140)

B $100

A

B $100

Transaction Date

When an entity maintaining books using US$ engages in a transaction in a foreign currency, they record the transaction using the spot rate (for currency exchange) as at the transaction date.

Accounts Receivable (2,000 X $1.43)

$2,860

Sales (2,000 X $1.43)

$2,860

Reporting Date

These need to be adjusted for changes in the spot rate as of the reporting date and the Foreign Exchange Gains/Losses are reported as Non-Operating Gains/Losses on the Income Statement.

Accounts Receivable (2,000 X $0.02)

$40

Unrealized Gain (2,000 X $0.02)

$40

Settlement Date

Upon settlement, a transaction gain or loss, measured from the transaction date or the most recent intervening Balance Sheet Date (whichever is later), should be included as a component of income from continuing operations for the period in which the transaction is settled.

Cash

$3,000

Accounts Receivable

$2,900

Unrealized Gain (2,000 X $0.05)

$100

Question # 164 | Blueprint Area: 1 A iii : Income Statement/Statement of Profit or Loss and Statement of Comprehensive IncomeRelated Chapter: FAR-11 : Foreign Currency Transactions

How well did you know this?
1
Not at all
2
3
4
5
Perfectly
2
Q

A partial listing of a company’s accounts is presented below:

Revenues $80,000
Operating Expenses $50,000
Foreign Currency Translation Adjustment Gain, Net of Tax $4,000
Income tax expense $10,000
What amount should the company report as net income?

B $24,000
C $30,000
D $34,000
A $20,000

A

A $20,000

The correct answer is (A).

Revenues $80,000 – Operating expenses $50,000 – Income tax expenses $10,000 = $20,000 Net income.

Foreign currency translation is the process of expressing a foreign entity’s functional currency financial statements in the reporting currency. Translation adjustments are included in the Cumulative Translation Adjustment (CTA) account, which is a component of Other Comprehensive Income (OCI). It is not included with net income.

Question # 171 | Blueprint Area: 1 A iii : Income Statement/Statement of Profit or Loss and Statement of Comprehensive IncomeRelated Chapter: FAR-2 : Income Statement and Statement of Comprehensive Income

How well did you know this?
1
Not at all
2
3
4
5
Perfectly
3
Q

Toigo Co. purchased merchandise from a vendor in England on November 20 for 500,000 British pounds. Payment was due in British pounds on January 20. The spot rates to purchase one pound were as follows:
November 20 $1.25
December 31 1.20
January 20 1.17How should the foreign currency transaction gain be reported on Toigo’s financial statements at December 31?
A A gain of $40,000 as a separate component of stockholders’ equity.
B A gain of $40,000 in the income statement.
C A gain of $25,000 as a separate component of stockholders’ equity.
D A gain of $25,000 in the income statement.

A

A gain of $25,000 in the income statement.

At the date the transaction is recognized, each asset, liability, revenue, expense, gain, or loss arising from the transaction should be measured and recorded in the functional currency of the recording entity by use of the exchange rate in effect at that date. Toiga would record a liability of $625,000 (500,000 x $1.25). At each balance sheet date, recorded balances that are denominated in a currency other than the functional currency of the recording entity should be adjusted to reflect the current exchange rate. These adjustments should be currently recognized as transaction gains or losses and reported as a component of income from continuing operations. Toigo would recognize a gain of $25,000 on the December 31 financial statements based on 500,000 pounds × $1.20 = $600,000 as of December 31.

Question # 165 | Blueprint Area: 1 A iii : Income Statement/Statement of Profit or Loss and Statement of Comprehensive IncomeRelated Chapter: FAR-11 : Foreign Currency Transactions

How well did you know this?
1
Not at all
2
3
4
5
Perfectly
4
Q

Which of the following statements is correct regarding reporting comprehensive income?
B A separate statement of comprehensive income is required.
C Comprehensive income must include all changes in stockholders’ equity for the period.
D Comprehensive income is reported in the year-end statements but not in the interim statements.

A Accumulated other comprehensive income is reported in the stockholders’ equity section of the balance sheet.

A

A Accumulated other comprehensive income is reported in the stockholders’ equity section of the balance sheet.

An entity is required to display the accumulated balance of other comprehensive income separately from retained earnings, capital stock, and additional paid-in capital in the stockholders’ equity section of a statement of financial position (balance sheet).

Question # 148 | Blueprint Area: 1 A iii : Income Statement/Statement of Profit or Loss and Statement of Comprehensive IncomeRelated Chapter: FAR-2 : Income Statement and Statement of Comprehensive Income

How well did you know this?
1
Not at all
2
3
4
5
Perfectly
5
Q

STUDY THIS

Which of the following is included in other comprehensive income?

A Unrealized holding gains and losses on trading debt securities.
B Unrealized holding gains and losses that result from debt security being transferred into the held-to-maturity category from the available-for-sale category.

D The difference between the accumulated benefit obligation and the fair value of pension plan assets.

C Foreign currency translation adjustments.

A

C Foreign currency translation adjustments.

An entity must classify items of other comprehensive income (OCI) by their nature. One of the items included in other comprehensive income is foreign currency translation gains and losses which result from the translation of financial statements of the subsidiary into reporting currency for consolidation with the parent.

Other Items included in OCI are

  • Unrealized holding gains and losses on available-for-sale (AFS) debt securities, not trading debt securities.
  • Unrealized holding gains and losses that result from debt security being transferred into the AFS category from the held-to-maturity (HTM) category and not to the HTM category from the AFS category.
  • For pensions, the amounts of net gain or loss, net prior service cost or credit, and net transition asset or obligation that are expected to be recognized as components of net periodic benefit cost; not the difference between the accumulated benefit obligation and the fair value of pension plan assets.
  • Effective and Ineffective Portion of Cashflow Hedge

Question # 139 | Blueprint Area: 1 A iii : Income Statement/Statement of Profit or Loss and Statement of Comprehensive IncomeRelated Chapter: FAR-2 : Income Statement and Statement of Comprehensive Income

How well did you know this?
1
Not at all
2
3
4
5
Perfectly
6
Q

Hail damaged several of Toncan Co.’s vans. Hailstorms had frequently inflicted similar damage to Toncan’s vans. Over the years, Toncan had saved money by not buying hail insurance and either paying for repairs, or selling damaged vans and then replacing them. The damaged vans were sold for less than their carrying amount. How should the hail damage cost be reported in Toncan’s financial statements?

A The actual hail damage loss as a component of other comprehensive income.

C The expected average hail damage loss in continuing operations, with no separate disclosure.
D The expected average hail damage loss in continuing operations, with separate disclosure.

B The actual hail damage loss in continuing operations, with no separate disclosure.

A

B The actual hail damage loss in continuing operations, with no separate disclosure.

The loss due to hailstorms should be reported in income from continuing operations. The full amount of a realized loss must be recognized in income when it occurs

Question # 101 | Blueprint Area: 1 A iii : Income Statement/Statement of Profit or Loss and Statement of Comprehensive IncomeRelated Chapter: FAR-2 : Income Statement and Statement of Comprehensive Income

How well did you know this?
1
Not at all
2
3
4
5
Perfectly
7
Q

On October 1 of the current year, Mild Co., a U.S. company, purchased machinery from Grund, a German company, with payment due on April 1 of next year. If Mild’s current-year operating income included no foreign exchange transaction gain or loss, then the transaction could have
A Resulted in a Foreign Currency translation gain to be reported in company’s income statement.

C Caused a foreign currency gain to be reported as a contra account against machinery.
D Caused a foreign currency translation gain to be reported in other comprehensive income.

B Been denominated in U.S. dollars.

A

Foreign currency transactions are transactions denominated in a currency other than the entity’s functional currency. Hence, no foreign currency transaction gain or loss would occur if the purchase of the machinery by the U.S. company is denominated in U.S. dollars.

Question # 160 | Blueprint Area: 1 A iii : Income Statement/Statement of Profit or Loss and Statement of Comprehensive IncomeRelated Chapter: FAR-11 : Foreign Currency Transactions

How well did you know this?
1
Not at all
2
3
4
5
Perfectly
8
Q

What is the purpose of reporting comprehensive income?

B To reconcile the difference between net income and cash flows provided from operating activities.
C To provide a consolidation of the income of the firm’s segments.
D To provide information for each segment of the business.

A To summarize all changes in equity from nonowner sources.

A

To summarize all changes in equity from nonowner sources.

Comprehensive income includes all the changes in equity during a period except those resulting from investments by owners & distribution to owners.

Option (B) is incorrect because it pertains to statement of cash flows.
Option (C) is incorrect because it pertains to segment reporting.
Option (D) is incorrect because it also pertains to segment reporting.

Question # 136 | Blueprint Area: 1 A iii : Income Statement/Statement of Profit or Loss and Statement of Comprehensive IncomeRelated Chapter: FAR-2 : Income Statement and Statement of Comprehensive Income

How well did you know this?
1
Not at all
2
3
4
5
Perfectly
9
Q

On January 1, Brecon Co. installed cabinets to display products in customers’ stores. Brecon expects to use these cabinets for five years. Brecon’s year-end multi-step income statement should include:

A One-fifth of the cabinet costs in cost of goods sold

C All of the cabinet costs in cost of goods sold
D All of the cabinet costs in selling, general, and administrative expenses

B One-fifth of the cabinet costs in selling, general, and administrative expenses

A

Brecon Co. will use the installed cabinets to display its merchandise in customers’ stores, which are fixed assets that would be used to generate day to day sales/revenue. The cost of installing the cabinets will be capitalized and depreciated over a period of 5 years. Depreciation is calculated on a straight-line basis for 5 years which means 1/5th of the cost per year will be charged to the income statement. The cabinets are used to display products for sale, depreciation will be reported under selling expense.

(A) is incorrect because the cabinets are used to display products for sale, so the depreciation i.e. one-fifth of the cabinet costs will be reported under selling expense and not in cost of goods sold.
(C) is incorrect because only the one-fifth cost will be charged to depreciation as part of selling expense and no cost relating to installation of cabinets will be reported in cost of goods sold.
(D) is incorrect because only the one-fifth cost will be charged to depreciation in the income statement under selling, general, and administrative expenses and not all of the cabinet installation cost.

Question # 130 | Blueprint Area: 1 A iii : Income Statement/Statement of Profit or Loss and Statement of Comprehensive IncomeRelated Chapter: FAR-2 : Income Statement and Statement of Comprehensive Income

How well did you know this?
1
Not at all
2
3
4
5
Perfectly
10
Q

Envoy Co. manufactures and sells household products. Envoy experienced losses associated with its small appliance group. Operations and cash flows for this group can be clearly distinguished from the rest of Envoy’s operations. Envoy plans to sell the small appliance group with its operations. What is the earliest point at which Envoy should report the small appliance group as a discontinued operation?

B When Envoy receives an offer for the segment
C When Envoy first sells any of the assets of the segment
D When Envoy sells the majority of the assets of the segment

A When Envoy classifies it as held for sale

A

The results of discontinued operations are reported separately from continuing operations. Discon­tinued operations refers to the operations of a component of an entity that has been disposed of or is still operat­ing, but is subject of a formal plan for disposal. A component of an entity is defined as a segment, reporting unit, or asset group whose operations and cash flows are clearly distinguished from the rest of the entity, operation­ally as well as for financial reporting purposes. The small appliance group clearly qualifies as a component of an entity. When Envoy classifies the small appliance group as held for sale it is in effect implementing a formal plan for disposal.

Options (B), (C) and (D) are incorrect because they do not satisfy both the conditions to be reported as discontinued operation.

Question # 115 | Blueprint Area: 1 A iii : Income Statement/Statement of Profit or Loss and Statement of Comprehensive IncomeRelated Chapter: FAR-2 : Income Statement and Statement of Comprehensive Income

How well did you know this?
1
Not at all
2
3
4
5
Perfectly
11
Q

Unusual or infrequent items are reported in the

A Income Statement
B Financial Statement Footnotes
D None of the Above
A transaction that is unusual in nature and/or infrequent in occurrence should be reported as a component of income from continuing operations.

C Income from Continuing Operations

A

A transaction that is unusual in nature and/or infrequent in occurrence should be reported as a component of income from continuing operations.

Question # 103 | Blueprint Area: 1 A iii : Income Statement/Statement of Profit or Loss and Statement of Comprehensive IncomeRelated Chapter: FAR-2 : Income Statement and Statement of Comprehensive Income

How well did you know this?
1
Not at all
2
3
4
5
Perfectly
12
Q

For a marketable debt securities portfolio classified as available-for-sale, which of the following amounts should be included in the period’s net income?

I. Unrealized holding losses during the period
II. Realized gains during the period
III. Changes in the Market Adjustment account during the period
A III only.

C I and II.
D I, II, and III.

B II only.

A

Unrealized holding gains and losses for AFS debt securities are excluded from earnings and reported in other comprehensive income until realized.

Any changes in the Market Adjustment account would have no effect on net income.

Question # 86 | Blueprint Area: 1 A iii : Income Statement/Statement of Profit or Loss and Statement of Comprehensive IncomeRelated Chapter: FAR-2 : Income Statement and Statement of Comprehensive Income

How well did you know this?
1
Not at all
2
3
4
5
Perfectly
13
Q

Currently reported net income Comprehensive income

FASB’s conceptual framework explains both financial and physical capital maintenance concepts. Which capital maintenance concept is applied to currently reported net income, and which is applied to comprehensive income?

A Financial capital Physical capital
B Physical capital Physical capital

D Physical capital Financial capital

C Financial capital Financial capital

A

The financial capital maintenance concept defines income as the change in net resources other than from owner transactions. The financial capital maintenance concept is the capital maintenance concept used in present financial statements and comprehensive income.

In contrast, under the physical capital maintenance concept, a return on physical capital results only if the physical productive capacity of the enterprise at the end of the period exceeds its capacity at the beginning of the period, also after excluding the effects of transactions with owners. The physical capital maintenance concept can be implemented only if inventories and property, plant, and equipment are measured by their current costs.

Question # 80 | Blueprint Area: 1 A iii : Income Statement/Statement of Profit or Loss and Statement of Comprehensive IncomeRelated Chapter: FAR-2 : Income Statement and Statement of Comprehensive Income

How well did you know this?
1
Not at all
2
3
4
5
Perfectly
14
Q

In open market transactions, Gold Corp. simultaneously sold its long-term investment in Iron Corp. bonds and purchased its own outstanding bonds. The broker remitted the net cash from the two transactions. Gold’s gain on the purchase of its own bonds exceeded its loss on the sale of the Iron bonds. Gold should report the

A Net effect of the two transactions as Other Comprehensive Income.
C Effect of its own bond transaction gain in income from continuing operations and report the Iron bond transaction as a loss from Discontinued Operations.
D Effect of its own bond transaction as an Income from Discontinued Operations, and report the Iron bond transaction loss in income from continuing operations.

B Net effect of the two transactions in income from continuing operations.

A

B Net effect of the two transactions in income from continuing operations.

Gains or losses from the sale of long-term investments are reported as income from continuing operations.

Question # 120 | Blueprint Area: 1 A iii : Income Statement/Statement of Profit or Loss and Statement of Comprehensive IncomeRelated Chapter: FAR-2 : Income Statement and Statement of Comprehensive Income

How well did you know this?
1
Not at all
2
3
4
5
Perfectly
15
Q

Upon the death of an officer, Jung Co. received the proceeds of a life insurance policy held by Jung on the officer. The proceeds were not taxable. The policy’s cash surrender value had been recorded on Jung’s books at the time of payment. What amount of revenue should Jung report in its statements?

A Proceeds received

C Proceeds received plus cash surrender value
D None

B Proceeds received less cash surrender value

A

Life insurance proceeds received on death of the officer will be treated as income only to the extent they exceed the cash surrender value of the policy. Cash surrender value of the policy is an asset that is recognized each time the life insurance premium increases the cash value of the policy. Jung Co. has a life insurance policy on an officer and recognized its cash surrender value as an asset at the time of payment of premium. On death of the officer, cash surrender value is recouped and the balance is treated as income in its financial statements.

Options (a), (c) and (d) are incorrect based on the above explanation.

Question # 91 | Blueprint Area: 1 A iii : Income Statement/Statement of Profit or Loss and Statement of Comprehensive IncomeRelated Chapter: FAR-2 : Income Statement and Statement of Comprehensive Income

How well did you know this?
1
Not at all
2
3
4
5
Perfectly
16
Q

On June 19, Don Co., a U.S. company, sold and delivered merchandise on a 30-day account to Cologne GmbH, a German corporation, for 200,000 euros. On July 19, Cologne paid Don in full. Relevant currency exchange rates were:
June 19 July 19
Spot rate $ .988 $ .995
30-day forward rate .990 1.000What amount should Don record on June 19 as an account receivable for its sale to Cologne?

B $198,000
C $199,000
D $200,000
A $197,600

A

At the date a transaction is recognized, each asset, liability, revenue, expense, gain, or loss arising from a foreign currency transaction should be measured and recorded in the functional currency of the recording entity by use of the exchange rate (i.e. spot rate) in effect at that date. Don Co should record an account receivable of $197,600 on June 19 (200,000 x .988)

Question # 166 | Blueprint Area: 1 A iii : Income Statement/Statement of Profit or Loss and Statement of Comprehensive IncomeRelated Chapter: FAR-11 : Foreign Currency Transactions

How well did you know this?
1
Not at all
2
3
4
5
Perfectly
17
Q

In Dart Co.’s year 2 single-step income statement, as prepared by Dart’s controller, the section titled “Revenues” consisted of the following:

Sales $250,000
Purchase discounts 3,000
Recovery of accounts written off 10,000
In its year 2 single-step income statement, what amount should Dart report as total revenues?

B $253,000
C $260,000
D $263,000
A $250,000

A

In the single step income statement, the Total revenues = Net sales + Other revenues & gains. Therefore, the Total revenue is $250,000.

Option (B) is incorrect because purchase discounts will not be included in the revenues but deducted from the cost of goods sold.
Option (C) is incorrect because recovery of accounts written off will not impact the revenues. The recovery will reduce the allowance for doubtful debts account.
Option (D) is incorrect as per the above explanation.

Question # 131 | Blueprint Area: 1 A iii : Income Statement/Statement of Profit or Loss and Statement of Comprehensive IncomeRelated Chapter: FAR-2 : Income Statement and Statement of Comprehensive Income

How well did you know this?
1
Not at all
2
3
4
5
Perfectly
18
Q

Net income Other comprehensive income

During the current year, Cooley Co. had an unrealized gain of $100,000 on a debt investment classified as available-for-sale. Cooley’s corporate tax rate is 25%. What amount of the gain should be included in Cooley’s net income and other comprehensive income at the end of the current year?

A $100,000 $0
B $75,000 $25,000
C $25,000 $75,000
D $0 $75,000

A

D $0 $75,000

Unrealized Gains and Losses on Certain Investments in debt securities including the following are reported exclusively as other comprehensive income:

Unrealized holding gains and losses on available-for-sale (AFS) debt securities.
Unrealized holding gains and losses that result from a debt security being transferred into the AFS category from the HTM category.
Subsequent decreases or increases in the fair value of AFS debt securities previously written down as impaired.
A change in the market value of a futures contract that qualifies as a hedge of an asset reported at fair value.
Such gains and losses are not classified as net income unless the securities are sold off.

Question # 175 | Blueprint Area: 1 A iii : Income Statement/Statement of Profit or Loss and Statement of Comprehensive IncomeRelated Chapter: FAR-2 : Income Statement and Statement of Comprehensive Income

How well did you know this?
1
Not at all
2
3
4
5
Perfectly
19
Q

A company reported the following information for year 1:

Net income $34,000
Owner contribution 9,000
Deferred gain on an effective cash-flow hedge 8,000
Foreign currency translation gain 2,000
Prior service cost not recognized in net periodic pension cost 5,000
What is the amount of other comprehensive income for year 1?

B $14,000
C $15,000
D $43,000

A $5,000

A

A $5,000

Other comprehensive income includes revenues, expenses, gains, and losses that are excluded from net income. Instead, they are listed after net income on the income statement. These items have not yet been realized. In other words, the underlying transaction has not been completed or settled yet. Based on this defi­nition, net income and any owner contribution would obviously not be included in other comprehensive income. Other comprehensive income items are classified by their nature within certain classifications such as: foreign currency items; pension adjustments, unrealized gains and losses on certain investments in debt and equity securities; and gains and losses on cash flow hedging derivative instruments. Other comprehensive income would include the gain on the effective cash-flow hedge, the foreign currency translation gain, and the prior service cost not recognized in net periodic pension cost.

Deferred gain on an effective cash-flow hedge $ 8,000
Foreign currency translation gain 2,000
Prior service cost not recognized in net periodic pension cost _(5,000)
Other comprehensive income for year 1 $ 5,000
Option (B), (C) and (D) are incorrect because the calculation above specifies the components for the other comprehensive income.

Question # 144 | Blueprint Area: 1 A iii : Income Statement/Statement of Profit or Loss and Statement of Comprehensive IncomeRelated Chapter: FAR-2 : Income Statement and Statement of Comprehensive Income

How well did you know this?
1
Not at all
2
3
4
5
Perfectly
20
Q

Fogg Co., a U.S. company, contracted to purchase foreign goods. Payment in foreign currency was due one month after the goods were received at Fogg’s warehouse. Between the receipt of goods and the time of payment, the exchange rates changed in Fogg’s favor. The resulting gain should be included in Fogg’s financial statements as a(an)

B Extraordinary item.
C Deferred credit.
D Item of other comprehensive income.
A Component of income from continuing operations.

A

A change in exchange rates between the functional currency and the currency in which the transaction is denominated increases or decreases the expected amount of functional currency cash flows upon a settlement of the transaction. That increase or decrease in expected functional currency cash flows is a foreign currency transaction gain or loss that generally should be included as a component of income from continuing operations for the period in which the transaction is settled.

Question # 162 | Blueprint Area: 1 A iii : Income Statement/Statement of Profit or Loss and Statement of Comprehensive IncomeRelated Chapter: FAR-11 : Foreign Currency Transactions

How well did you know this?
1
Not at all
2
3
4
5
Perfectly
21
Q

Which of the following describes how comprehensive income should be reported?

A Must be reported in a separate statement, as part of a complete set of financial statements
B Should not be reported in the financial statements but should only be disclosed in the footnotes

D May be reported in a combined statement of income and comprehensive income or disclosed within a statement of stockholders’ equity; separate statements of comprehensive income are not permitted

C May be reported in a separate statement or in a combined statement of income and comprehensive income

A

An entity may choose from two possible formats to report comprehensive income. The income statement and the statement of comprehensive income are separate statements in the two-statement format. Comprehensive income must be displayed prominently within a financial statement in a full set of general purpose financial statements. Comprehensive income must be shown on the face of one of the statements, not just in the notes to the financial statements.

Question # 147 | Blueprint Area: 1 A iii : Income Statement/Statement of Profit or Loss and Statement of Comprehensive IncomeRelated Chapter: FAR-2 : Income Statement and Statement of Comprehensive Income

How well did you know this?
1
Not at all
2
3
4
5
Perfectly
22
Q

Year 3 Year 4

On October 1, year 3, Wand, Inc. committed itself to a formal plan to sell its Kam division’s assets. Wand estimated that the loss from the disposal of assets in February of year 4 would be $25,000. Wand also estimated that Kam would incur operating losses of $100,000 for the period of October 1 through December 31, year 3, and $50,000 for the period January through February 28, year 4. These estimates were materially correct. Disregarding income taxes, what should Wand report as loss from discontinued operations in its comparative year 3 and year 4 income statements?

A $175,000 $0

C $100,000 $ 75,000
D $0 $175,000

B $125,000 $ 50,000

A

B $125,000 $ 50,000
The income statement of a business enterprise for current and prior periods shall report the results of operations of the component in discontinued operations in the period(s) in which they occur. The impairment loss should be recognized in the period in which a sale at a loss is arranged; such a sale is evidence that the asset is impaired.

Question # 110 | Blueprint Area: 1 A iii : Income Statement/Statement of Profit or Loss and Statement of Comprehensive IncomeRelated Chapter: FAR-2 : Income Statement and Statement of Comprehensive Income

How well did you know this?
1
Not at all
2
3
4
5
Perfectly
23
Q

Green Corp. owns 30% of the outstanding common stock and 100% of the outstanding noncumulative nonvoting preferred stock of Axel Corp. In the current year, Axel declared dividends of $100,000 on its common stock and $60,000 on its preferred stock. Green exercises significant influence over Axel’s operations.

What amount of dividend revenue should Green report in its income statement for the current year ended December 31?

A $0
B $30,000

D $90,000

C $60,000

A

C $60,000

The investment in the nonvoting preferred stock must be accounted under the cost method because Green does not have the ability to significantly influence the financial and operating policies of the investee by virtue of the preferred stock investment. Therefore, the $60,000 dividend declared on the preferred stock is reported as dividend revenue by Green. The 30% investment in the common stock should be accounted for under the equity method because Green has the ability to exercise significant influence over the investee by virtue of the investment. Under the equity method, the investor recognizes as income its share of the investor’s earnings in the periods in which they are reported by the investee. These amounts are recognized as equity in the earnings of the investee and not as dividend revenue.

Question # 167 | Blueprint Area: 1 A iii : Income Statement/Statement of Profit or Loss and Statement of Comprehensive IncomeRelated Chapter: FAR-2 : Income Statement and Statement of Comprehensive Income

How well did you know this?
1
Not at all
2
3
4
5
Perfectly
24
Q

A company reported the following for the current year:

Retained earnings appropriated for plant expansion $32,500
Correction of understated depreciation expense from prior periods $9,300
Unrealized loss on available-for-sale debt securities $8,100
Unrealized gain on foreign currency translation $3,400
The company’s current-year net income was $86,500, and the company has a 30% effective income tax rate. What amount of comprehensive income should be reported for the current year?

A $40,000
B $76,700
C $81,800
D $83,210

A

D $83,210

Question # 181 | Blueprint Area: 1 A iii : Income Statement/Statement of Profit or Loss and Statement of Comprehensive IncomeRelated Chapter: FAR-2 : Income Statement and Statement of Comprehensive Income

How well did you know this?
1
Not at all
2
3
4
5
Perfectly
25
Q

In year 1, Chain, Inc. purchased a $1,000,000 life insurance policy on its president, of which Chain is the beneficiary. Information regarding the policy for the year ended December 31, Year 6, follows:

Cash surrender value, 1/1, year 6 $ 87,000
Cash surrender value, 12/31, year 6 108,000
Annual advance premium paid, 1/1, year 6 40,000
During year 6, dividends of $6,000 were applied to increase the cash surrender value of the policy. What amount should Chain report as life insurance expense for year 6?

A $40,000
B $25,000
C $19,000
D $13,000

A

C $19,000

The purchase of life insurance on an officer can be a form of investment when it builds up a cash value (cash surrender value). Cash surrender value is the amount an insurance company pays to the policyholder in the event the policy is surrendered before maturity. The portion of the premium that increases the cash value is accumulated as a non-current asset on the balance sheet and the rest of the premium is recognized as life insurance expense.

Chain is the holder of a $1,000,000 life insurance policy whose cash surrender value was $87,000 on January 1, 20X1 and $108,000 on December 31,20X1, an increase of$21,000 during the year 20X1. Advance premium paid is $40,000. Out of this premium payment, $21,000 would be recorded as an increase in the cash surrender value of the policy. The balance $19,000 (i.e. premium $40,000 – increase in CSV $21,000) will be recorded as insurance expense for 20X1.

Dividend of $6,000 applied to increase the cash surrender value of the policy, is an allocation done by the insurance company. To Chain, the policyholder, this gets reflected as part of the $21,000 increase in CSV of the policy during 20X1. This dividend is not an income to Chain and is ignored.

Option (A) is incorrect because $40,000 is the entire premium paid on the policy. Of this, $21,000 would be recorded towards increase in cash surrender value. The balance $19,000 would be the insurance expense for the year. Option (B) is incorrect because $25,000 insurance expense is arrived by reducing the increase in CSV during the year by the dividend of $6,000. Premium paid – (increase in CSV of policy – dividend received) = $40,000 – ($21,000 - $6,000) = $25,000.Option (D) is incorrect because $13,000 insurance expense is arrived by considering dividend of $6,000 as income to Chain and by applying it to reduce the expense (i.e. Premium paid – Increase in CSV – Dividend = $40,000 - $21,000 - $6,000 = $13,000).

Question # 89 | Blueprint Area: 1 A iii : Income Statement/Statement of Profit or Loss and Statement of Comprehensive IncomeRelated Chapter: FAR-2 : Income Statement and Statement of Comprehensive Income

How well did you know this?
1
Not at all
2
3
4
5
Perfectly
26
Q

A company reports the following information as of December 31:

Sales revenue $800,000
Cost of goods sold 600,000
Operating expenses 110,000
Foreign Currency Translation Gain 25,000
Non-Operating Gain 35,000
Ignoring Income Taxes, what amount should the company report as comprehensive income as of December 31?

A $90,000
B $125,000

D $115,000

C $150,000

A

C $150,000

The comprehensive income items will be presented in the following manner:

Sales $800,000
Cost of goods sold (600,000)
Operating expenses (110,000)
Non-Operating Gain 35,000
Foreign currency translation gain (part of OCI) 25,000
Total $150,000
Option (A) is incorrect because it is the income from continuing operations.
Option (B) is incorrect because it is the net income.
Option (D) is incorrect because it does not include the non-operating gain.

Question # 138 | Blueprint Area: 1 A iii : Income Statement/Statement of Profit or Loss and Statement of Comprehensive IncomeRelated Chapter: FAR-2 : Income Statement and Statement of Comprehensive Income

How well did you know this?
1
Not at all
2
3
4
5
Perfectly
27
Q

Kent Co. incurred the following infrequent losses during the year:

A $300,000 loss was incurred on disposal of one of four dissimilar factories.
$120,000 foreign exchange translation loss.
Inventory valued at $190,000 was made worthless by a competitor’s unexpected product innovation.
In its year-end income statement, what amount should Kent report as income (loss) from continuing operations?

A ($610,000)
B ($490,000)

D ($310,000)

C ($190,000)

A

The loss on disposal of one of four dissimilar factories of $300,000 is a discontinued operations item and would not be reported in continuing operations.

A foreign currency translation loss of $120,000 would be reported in Other Comprehensive Income.

Income (Loss) from Continuing Operations will include only the inventory obsolescence loss of $190,000.

Question # 98 | Blueprint Area: 1 A iii : Income Statement/Statement of Profit or Loss and Statement of Comprehensive IncomeRelated Chapter: FAR-2 : Income Statement and Statement of Comprehensive Income

How well did you know this?
1
Not at all
2
3
4
5
Perfectly
28
Q

During the current year, Onal Co. purchased 10,000 shares of its own stock at $7 per share. The stock was originally issued at $6. The firm sold 5,000 of the treasury shares for $10 per share. The firm uses the cost method to account for treasury stock. What amount should Onal report in its income statement for these transactions?

B $ 5,000 gain.
C $10,000 loss.
D $15,000 gain.

A $0.

A

Treasury stock is the corporation’s common or preferred stock that has been reacquired by purchase, by the settlement of an obligation to the corporation, or through donation. The cost method views the purchase and subsequent disposition of stock as one transaction. The treasury stock is recorded (debited), carried, and reissued at the acquisition cost. If the stock is reissued at a price in excess of the acquisition cost, such as in this situation, the excess is credited to an appropriately titled paid-in capital account. There are no gains or losses reported in the income statement for these transactions.

The journal entries are as follows:

Repurchased 10,000 Shares at $7 per share:

Dr. Treasury Stock $70,000

Cr. Cash $70,000

Reissued 5,000 Shares at $10 per share:

Dr. Cash $50,000

Cr. Treasury Stock $35,000

Cr. Additional Paid-in-Capital – Treasury Stock $15,000

Question # 94 | Blueprint Area: 1 A iii : Income Statement/Statement of Profit or Loss and Statement of Comprehensive IncomeRelated Chapter: FAR-2 : Income Statement and Statement of Comprehensive Income

How well did you know this?
1
Not at all
2
3
4
5
Perfectly
29
Q

Ocean Corp.’s comprehensive insurance policy allows its assets to be replaced at current value. The policy has a $50,000 deductible clause. One of Ocean’s waterfront warehouses was destroyed in a winter storm. Such storms occur approximately every four years. Ocean incurred $20,000 of costs in dismantling the warehouse and plans to replace it. The following data relate to the warehouse:

Current carrying amount $ 300,000
Replacement cost 1,100,000
What amount of gain should Ocean report as a separate component of income on the income statement?

A $1,030,000
B $ 780,000

D $ 0

C $ 730,000

A

C $ 730,000

A gain or loss on the involuntary conversion (e.g., casualty, condemnation, theft) of a nonmonetary asset is recognized in income even if the proceeds received as a result of the involuntary conversion are reinvested in a replacement nonmonetary asset. The gain on the involuntary conversion is reported as a separate component of income from continuing operations.

Question # 127 | Blueprint Area: 1 A iii : Income Statement/Statement of Profit or Loss and Statement of Comprehensive IncomeRelated Chapter: FAR-2 : Income Statement and Statement of Comprehensive Income

How well did you know this?
1
Not at all
2
3
4
5
Perfectly
30
Q

The presentation of income from continuing operations in a _______________ emphasizes a functional or object classification of each statement item, and sets forth various intermediate levels of income.
A Comparative statement format

C Single-step format
D None of the above

B Multiple-step format

A

B Multiple-step format

The presentation of income from continuing operations in a multiple-step format (not comparative statement format) emphasizes a functional or object classification of each statement item, and sets forth various intermediate levels of income.

Question # 133 | Blueprint Area: 1 A iii : Income Statement/Statement of Profit or Loss and Statement of Comprehensive IncomeRelated Chapter: FAR-2 : Income Statement and Statement of Comprehensive Income

How well did you know this?
1
Not at all
2
3
4
5
Perfectly
31
Q

Which of the following items is not classified as “other comprehensive income”?

B Foreign currency translation adjustments
C Additional pension liability adjustment for a defined-benefit pension plan
D Unrealized gains for the year on available-for-sale debt securities

A Gains from extinguishment of debt

A

A Gains from extinguishment of debt

Gains from extinguishment of debt are not classified as Other Comprehensive Income.

Other Comprehensive Income includes

  • Foreign currency translation adjustments
  • Additional pension liability adjustments for a defined-benefit pension plan
  • Adjustments for unrealized gains and losses on available-for-sale debt securities
  • Effective Cashflow Hedges

Question # 84 | Blueprint Area: 1 A iii : Income Statement/Statement of Profit or Loss and Statement of Comprehensive IncomeRelated Chapter: FAR-2 : Income Statement and Statement of Comprehensive Income

How well did you know this?
1
Not at all
2
3
4
5
Perfectly
32
Q

For the last 10 years, Woody Co. has owned cumulative preferred stock issued by Hadley, Inc. During year 12, Hadley declared and paid both the year 12 dividend and the year 11 dividend in arrears. How should Woody report the year 11 dividend in arrears that was received in year 12?

A As a reduction in cumulative preferred dividends receivable
B As a retroactive change of the prior period financial statements
C Include, net of income taxes, after year 12 income from continuing operations
D Include in year 12 income from continuing operations

A

D Include in year 12 income from continuing operations

The preferred stock investment should be accounted for under the FVTNI method (Fair Value through Net Income) since it is highly unlikely that the investor has the ability to exercise significant influence over the operating and financial policies of the investee by virtue of the investment.

Preferred stock is usually nonvoting.

Under the FVTNI method, dividends are normally not recognized as income until they are declared by the investee.

By definition, dividends in arrears have not yet been declared. Dividend income is a component of income from continuing operations.

Question # 99 | Blueprint Area: 1 A iii : Income Statement/Statement of Profit or Loss and Statement of Comprehensive IncomeRelated Chapter: FAR-2 : Income Statement and Statement of Comprehensive Income

How well did you know this?
1
Not at all
2
3
4
5
Perfectly
33
Q

If Tron Inc. a US-based entity has accounts payable valued in foreign currency, it should be adjusted for changes in the:

A Spot rate and the foreign exchange gains or losses are reported as operating gains or losses on the income statement.

C Average rate and the foreign exchange gains or losses are reported as operating gains or losses on the income statement.
D Average rate and the foreign exchange gains or losses are reported as non-operating gains or losses on the income statement.

B Spot rate and the foreign exchange gains or losses are reported as non-operating gains or losses on the income statement.

A

B Spot rate and the foreign exchange gains or losses are reported as non-operating gains or losses on the income statement.

When an entity with the US Dollar as a functional currency engages in a transaction in a foreign currency, it remeasures the transaction using the spot rate as of the transaction date.

If the entity has any monetary assets or liabilities (e.g., cash, accounts receivable, accounts payable) valued in the foreign currency, these need to be adjusted for changes in the spot rate and the foreign exchange gains or losses are reported as non-operating gains or losses on the Income Statement.

Question # 158 | Blueprint Area: 1 A iii : Income Statement/Statement of Profit or Loss and Statement of Comprehensive IncomeRelated Chapter: FAR-11 : Foreign Currency Transactions

How well did you know this?
1
Not at all
2
3
4
5
Perfectly
34
Q

Jordan Co. had the following gains during the current period:

Gain on disposal of a business segment $500,000
Foreign currency translation gain 100,000
What amount of gains should be presented on Jordan’s income statement as income from continuing operations?

B $100,000
C $500,000
D $600,000
A $0

A

A $0

Gain on disposal of business segments will be reported as discontinued operations and foreign currency translation gains will be reported in other comprehensive income. Therefore, neither of the items will be reflected in income from continuing operations.

Option (B) is incorrect because it will be a part of other comprehensive income.
Option (C) is incorrect because it should be reported as a discontinued operation.
Option (D) is incorrect as per the above explanation.

Question # 124 | Blueprint Area: 1 A iii : Income Statement/Statement of Profit or Loss and Statement of Comprehensive IncomeRelated Chapter: FAR-2 : Income Statement and Statement of Comprehensive Income

How well did you know this?
1
Not at all
2
3
4
5
Perfectly
35
Q

On September 30, year 1, a commitment was made to dispose of a business segment in early year 2. The segment operating loss for the period October 1 to December 31, year 1, should be included in the year 1 income statement as part of

A Loss on disposal of the discontinued segment.

C Income or loss from continuing operations.
D Other Comprehensive Income.
B Operating loss of the discontinued segment.

A

B Operating loss of the discontinued segment.

In the period in which a component of an entity either has been disposed of or is classified as held for sale, the income statement of a business enterprise for current and prior periods shall report the results of operations of the component in discontinued operations in the period(s) in which they occur.

The segment operating loss for the period October 1 to December 31, year 1, would be included in the year 1 income statement discontinued operations section as part of the operating loss of the discontinued segment.

Question # 150 | Blueprint Area: 1 A iii : Income Statement/Statement of Profit or Loss and Statement of Comprehensive IncomeRelated Chapter: FAR-2 : Income Statement and Statement of Comprehensive Income

How well did you know this?
1
Not at all
2
3
4
5
Perfectly
36
Q

Which of the following statements regarding discontinued operations items is true?

A A discontinued operations item should be classified separately in the income statement net of taxes.
B Unusual or infrequent items should be classified as discontinued operations.
C If the item is determined to be from discontinued operations, it should be presented separately on the income statement gross of related income tax.
D Gains on the disposal of a segment of a business should not be reported as discontinued operation items.

A A discontinued operations item should be classified separately in the income statement net of taxes.

A

A A discontinued operations item should be classified separately in the income statement net of taxes.

A component of an entity that has been disposed of or has been classified as held for sale is reported as a discontinued operations item that should be classified separately in the income statement net of taxes after income (loss) from continuing operations.

Question # 125 | Blueprint Area: 1 A iii : Income Statement/Statement of Profit or Loss and Statement of Comprehensive IncomeRelated Chapter: FAR-2 : Income Statement and Statement of Comprehensive Income

How well did you know this?
1
Not at all
2
3
4
5
Perfectly
37
Q

Vane Co.’s trial balance of income statement accounts for the year ended December 31, included the following:

B $215,000
C $280,000
D $295,000

A $200,000

A

A $200,000

Cost of Goods Manufactured is calculated as follows:

Opening Inventory + Cost of Goods Manufactured – Ending Inventory = Cost of Goods Sold
$400,000 + Cost of Goods Manufactured - $360,000 = $240,000
Cost of Goods Manufactured = $240,000 + $360,000 - $400,000
Cost of Goods Manufactured = $600,000 - $400,000
Cost of Goods Manufactured = $200,000

Question # 128 | Blueprint Area: 1 A iii : Income Statement/Statement of Profit or Loss and Statement of Comprehensive IncomeRelated Chapter: FAR-2 : Income Statement and Statement of Comprehensive Income

How well did you know this?
1
Not at all
2
3
4
5
Perfectly
38
Q

Year 5 Year 4

During January of Year 4, Doe Corp. agreed to sell the assets and product line of its Hart divi­sion. The sale on January 15 of Year 5 resulted in a gain on disposal of $900,000. Not consider­ing any impairment losses, Hart’s operating losses were $600,000 for Year 4 and $50,000 for the period of January 1 through January 15 of Year 5. Disregarding income taxes, what amount of net gain (loss) should be reported in Doe’s comparative Year 4 and 5 income statements?

A $0 $250,000
B $250,000 $0
D $900,000 $(650,000)
C $850,000 $(600,000)

A

C $850,000 $(600,000)

The income statement of a business enterprise for current and prior periods shall report the results of operations of the component in discontinued operations in the period(s) in which they occur.

Question # 112 | Blueprint Area: 1 A iii : Income Statement/Statement of Profit or Loss and Statement of Comprehensive IncomeRelated Chapter: FAR-2 : Income Statement and Statement of Comprehensive Income

How well did you know this?
1
Not at all
2
3
4
5
Perfectly
39
Q

On September 22 of the previous year, Yumi Corp. purchased merchandise from an unaffiliated foreign company for 10,000 units of the foreign company’s local currency. On that date, the spot rate was $.55. Yumi paid the bill in full on March 20 of the current year, when the spot rate was $.65. The spot rate was $.70 on December 31 of the previous year. What amount should Yumi report as a foreign currency transaction loss in its income statement for the previous year ended December 31?
A $0
B $ 500
C $1,000
D $1,500

A

D $1,500

Whenever a transaction is denominated (i.e., payable) in a foreign currency, changes in the translation rate of the foreign currency with respect to the entity’s functional currency (i.e., the dollar in this case) will result in a transaction gain or loss. The gain or loss should be included in the determination of net income in the period(s) the rate changes, and the related asset or liability (i.e., accounts payable in this case) should be adjusted accordingly.
Accounts payable, 12/31 of previous year, in $US

(10,000 local currency units × $0.70) $7,000
Initial obligation, 9/22 or previous year, in $US

(10,000 local currency units × $0.55) (5,500)
Foreign exchange loss recognized in the previous year $1,500

Question # 161 | Blueprint Area: 1 A iii : Income Statement/Statement of Profit or Loss and Statement of Comprehensive IncomeRelated Chapter: FAR-11 : Foreign Currency Transactions

How well did you know this?
1
Not at all
2
3
4
5
Perfectly
40
Q

Burns Corp. had the following items:

Sales revenue $45,000
Loss on early extinguishment of bonds 36,000
Realized gain on sale of available-for-sale debt securities 28,000
Unrealized holding loss on available-for-sale debt securities 17,000
Loss on write-down of inventory 3,100
Which of the following amounts would the statement of comprehensive income report as other comprehensive income or loss?

A $11,000 other comprehensive income.
B $16,900 other comprehensive income.

D $28,100 other comprehensive loss.

C $17,000 other comprehensive loss.

A

C $17,000 other comprehensive loss.

Sales revenue is operating income item. The loss on early extinguishment of bonds, realized gain on sale of available-for-sale debt securities and the loss on write down of inventory are the other revenue/gain and other expense/loss items. Only $17,000 unrealized gain/loss on available-for-sale debt securities is included in the other comprehensive income.

Option (A) is incorrect because it includes the realized gain on sales of available-for sale debt securities.
Option (B) and (D) are incorrect because they each include components of net income.

Question # 142 | Blueprint Area: 1 A iii : Income Statement/Statement of Profit or Loss and Statement of Comprehensive IncomeRelated Chapter: FAR-2 : Income Statement and Statement of Comprehensive Income

How well did you know this?
1
Not at all
2
3
4
5
Perfectly
41
Q

A U.S. company purchased inventory on account at a cost of 1,000 foreign currency units (FCU) from a non-U.S. company on November 15, to be paid on December 15. The FCU is valued at $0.85 on November 15 and at $0.90 on December 15. The journal entry to record payment on December 15 should include which of the following?

A Debit inventory and credit cash for $850

C Debit accounts payable and credit exchange gains and losses for $50
D Debit accounts payable and credit cash for $850

B Debit exchange gains and losses and credit accounts payable for $50

A

B Debit exchange gains and losses and credit accounts payable for $50

Purchased inventory would have been recorded for $850 on November 15th because of $0.85 FCU value x 1,000. On December 15th, with an exchange rate of $0.90, it will require $900 to make the payment. Accounts payable will be increased with a credit of $50 and an exchange loss will be recognized with a debit of $50.

Exchange loss $50
A/P $50
(A) is incorrect because inventory will be debited at the purchase date

(C) is incorrect because there is a loss, the reverse of this entry must be passed.

(D) is incorrect because the amount should include exchange gain or loss

Question # 170 | Blueprint Area: 1 A iii : Income Statement/Statement of Profit or Loss and Statement of Comprehensive IncomeRelated Chapter: FAR-11 : Foreign Currency Transactions

How well did you know this?
1
Not at all
2
3
4
5
Perfectly
42
Q

On April 30, Deer approved a plan to dispose of a segment of its business. For the period January 1 through April 30, the segment had revenues of $500,000 and expenses of $800,000. The assets of the seg­ment were sold on October 15, at a loss for which no tax benefit is available. In its income statement for the calendar year, how should Deer report the segment’s operations from January 1 to April 30?

A $500,000 and $800,000 included with revenues and expenses, respectively, as part of continuing operations.
B $300,000 reported as a net loss, as part of continuing operations.
C $300,000 reported as Other Comprehensive Income.
D $300,000 reported as a loss from discontinued operations.

A

D $300,000 reported as a loss from discontinued operations.

In the case of discontinued operations once the plan is approved for disposal, the income or loss from operations from the beginning of the period to the date on which the decision was made is reported as a gain or loss from operations of a discontinued segment. In this case, for the period January 1 through April 30, the segment had revenues of $500,000 and expenses of $800,000 i.e., loss of $300,000 would be reported by Deer as part of discontinued operations.

Question # 113 | Blueprint Area: 1 A iii : Income Statement/Statement of Profit or Loss and Statement of Comprehensive IncomeRelated Chapter: FAR-2 : Income Statement and Statement of Comprehensive Income

43
Q

On October 1, year 4, Host Co. approved a plan to dispose of a segment of its business. Host expected that the sale would occur on April 1, year 5, at an estimated gain of $350,000. The segment had actual and estimated operating losses as follows:

1/1 to 9/30, year 4 $(300,000)
10/1 to 12/31, year 4 (200,000)
1/1 to 3/31, year 5 (400,000)
In its December 31, year 4 income statement, what should Host report as a loss from discontinued operations before income taxes?

A $200,000
C $250,000
D $600,000
C $500,000

A

C $500,000

The income statement of a business enterprise for current and prior periods shall report the results of operations of the component in discontinued operations in the period(s) in which they occur. The loss on disposal of the segment in year 4 is $500,000 [$300,000 + $200,000].

Question # 111 | Blueprint Area: 1 A iii : Income Statement/Statement of Profit or Loss and Statement of Comprehensive IncomeRelated Chapter: FAR-2 : Income Statement and Statement of Comprehensive Income

44
Q

An increase in the cash surrender value of a life insurance policy owned by a company would be recorded by

B Increasing investment income.
C Recording a memorandum entry only.
D Decreasing a deferred charge.
A Decreasing annual insurance expense.

A

A Decreasing annual insurance expense.

The amount that a company should recognize as insurance expense for a life insurance policy that it owns is the annual premium less (1) the increase in cash surrender value and (2) any dividends received.

Question # 90 | Blueprint Area: 1 A iii : Income Statement/Statement of Profit or Loss and Statement of Comprehensive IncomeRelated Chapter: FAR-2 : Income Statement and Statement of Comprehensive Income

45
Q

Sales Sales tax payable

A customer is considering buying a television set with a retail price of $2,000. The customer asks the store manager if the store will consider paying the sales tax so that the total cash payment is $2,000. The sales tax is 8%. The store manager agrees to accept $2,000 cash. What should the accountant credit in this transaction?

A $2,000 $0
B $1,840 $160
C $2,000 $148
D $1,852 $148

A

D $1,852 $148

Normally, the customer would have paid $2,000 for the television and 8% of $2,000 as sales tax to the store manager. However, in this case, the customer requests the store manager to take a final payment of $2,000. This implies that this amount would include the sales tax payable, causing a reduction in the income of the store.
Sales tax included in final payment = $2,000 x 8%/108% = $148
The journal entry to account for this would be as follows:

Cash $2,000
Sales $1,852
Sales tax payable $148
The portion of sales tax is a liability for the store as it is to be remitted to the government.

Question # 177 | Blueprint Area: 1 A iii : Income Statement/Statement of Profit or Loss and Statement of Comprehensive IncomeRelated Chapter: FAR-2 : Income Statement and Statement of Comprehensive Income

46
Q

Which of the following components would not form a part of comprehensive income?

A Unrealized gains on available-for-sale securities
B Gain on disposal of a fixed asset

D Loss on disposal of a segment

C Cash dividends paid to common shareholders

A

C Cash dividends paid to common shareholders

Comprehensive income is the sum of net income and other comprehensive income. It aims to show all the components that change the equity except for owner transactions. All non-owner transactions are included in the comprehensive income. Cash dividends paid to common shareholders are an example of an owner transaction, and as such, it wouldn’t form part of the comprehensive income of the organization.

Question # 180 | Blueprint Area: 1 A iii : Income Statement/Statement of Profit or Loss and Statement of Comprehensive IncomeRelated Chapter: FAR-2 : Income Statement and Statement of Comprehensive Income

47
Q

Unusual or infrequent items are reported:

A Extraordinary item, net of applicable income taxes.
B Extraordinary item, but not net of applicable income taxes.

D Component of income from continuing operations, net of applicable income taxes.

C Component of income from continuing operations, but not net of applicable income taxes.

A

C Component of income from continuing operations, but not net of applicable income taxes.

A transaction that is unusual in nature or infrequent in occurrence should be reported as a component of income from continuing operations. It cannot be shown as net of taxes as tax is a separate line item to arrive at the income from continuing operations.

Option (A) and (B) are incorrect because normally the unusual and infrequent gains or losses reported as extraordinary item. With effect from December 2015, the Financial Accounting Standards Board (FASB) has eliminated the concept of extraordinary items from GAAP. Option (D) is incorrect because component of income from continuing operations are not shown net of taxes.

Question # 104 | Blueprint Area: 1 A iii : Income Statement/Statement of Profit or Loss and Statement of Comprehensive IncomeRelated Chapter: FAR-2 : Income Statement and Statement of Comprehensive Income

48
Q

Comprehensive Income excludes changes in equity resulting from which of the following?

A Loss from discontinued operations
B Prior period error correction

D Unrealized loss on investments in non-current marketable equity securities

C Dividends paid to stockholders

A

C Dividends paid to stockholders

Comprehensive income includes all changes in equity except those caused by owner investments or distributions to owners. Loss from discontinued operations is included in net income (a component of comprehensive income). Prior period error correction is a change in stockholders’ equity. Unrealized loss on investments in non-current marketable equity securities is a change in stockholders’ equity.

Question # 83 | Blueprint Area: 1 A iii : Income Statement/Statement of Profit or Loss and Statement of Comprehensive IncomeRelated Chapter: FAR-2 : Income Statement and Statement of Comprehensive Income

49
Q

Raim Cane

During the current year, both Raim Co. and Cane Co. suffered losses due to the flooding of the Mississippi River. Raim is located two miles from the river and sustains flood losses every two to three years. Cane, which has been located fifty miles from the river for the past twenty years, has never before had flood losses. How should the flood losses be reported in each company’s year-end income statement?

B As an extraordinary item, net of tax As a component of income from continuing operations
C As an extraordinary item, pre-tax As a component of income from continuing operations
D As a component of income from continuing operations or in the Financial Statement Notes As a component of income from continuing operations or in the Financial Statement Notes

A As a component of income from continuing operations As a component of income from continuing operations

A

A As a component of income from continuing operations As a component of income from continuing operations

A transaction that is unusual in nature and/or infrequent in occurrence should be reported as a component of income from continuing operations.

A flood loss for Raim Co., which is located two miles from the river and sustains flood losses every two to three years, is unusual but not infrequent and would be reported as a component of income from continuing operations.

A flood loss for Cane Co., which is located fifty miles from the river, and for the past twenty years has never before had flood losses, is both unusual and infrequent and would be reported as a component of income from continuing operations.

Question # 121 | Blueprint Area: 1 A iii : Income Statement/Statement of Profit or Loss and Statement of Comprehensive IncomeRelated Chapter: FAR-2 : Income Statement and Statement of Comprehensive Income

50
Q

The following costs were incurred by Griff Co., a manufacturer, during the year:

Accounting and legal fees $ 25,000
Freight-in 175,000
Freight-out 160,000
Officers’ salaries 150,000
Insurance 85,000
Sales representatives’ salaries 215,000
What amount of these costs should be reported as general and administrative expenses for the year?

B $550,000
C $635,000
D $810,000

A $260,000

A

A $260,000

The freight-in cost is an inventoriable cost. The freight-out cost and the sales representatives’ sala­ries should both be reported as selling expenses in the year.

Accounting and legal fees $ 25,000
Officers’ salaries 150,000
Insurance _ 85,000
General and administrative expenses $260,000

Question # 102 | Blueprint Area: 1 A iii : Income Statement/Statement of Profit or Loss and Statement of Comprehensive IncomeRelated Chapter: FAR-2 : Income Statement and Statement of Comprehensive Income

51
Q

When a full set of general-purpose financial statements is presented, comprehensive income and its components should

A Appear as a part of discontinued operations.
B Be reported net of related income tax effect, in total and individually.
C Appear in a supplemental schedule in the notes to the financial statements.
D Be displayed in a financial statement that has the same prominence as other financial statements.

A

D Be displayed in a financial statement that has the same prominence as other financial statements.

Comprehensive income and all items that are required to be recognized as components of comprehensive income should be reported in a financial statement that is displayed with the same prominence as other financial statements.

Discontinued operations are a component of the income statement, reported after income from continuing operations and before net income. If comprehensive income is reported in the same statement as net income, other comprehensive income (OCI) and comprehensive income are reported after net income. An entity may display components of OCI in either net-of-tax basis or summary net-of-tax basis.

Comprehensive income must be shown on the face of one of the statements, not just in the notes to the financial statements.

Question # 146 | Blueprint Area: 1 A iii : Income Statement/Statement of Profit or Loss and Statement of Comprehensive IncomeRelated Chapter: FAR-2 : Income Statement and Statement of Comprehensive Income

52
Q

Which of the following items should be shown as a component of comprehensive income?
A Dividend paid to a shareholder

C Additional capital contribution
D Deferred revenue

B Foreign-currency translation adjustment

A

B Foreign-currency translation adjustment

Comprehensive income is divided into net income and other comprehensive income (OCI). OCI includes revenues, expenses, gains, and losses that are excluded from net income. An entity must classify items of other comprehensive income by their nature, in one of these classifications: foreign currency items, pension adjustments, unrealized gains and losses on certain investments in debt and equity securities, and gains and losses on cash flow hedging derivative instruments. Included in foreign currency items are foreign currency translation adjustments. Dividends paid to a shareholder, additional capital contribution, and deferred revenue are not components of comprehensive income.

Question # 145 | Blueprint Area: 1 A iii : Income Statement/Statement of Profit or Loss and Statement of Comprehensive IncomeRelated Chapter: FAR-2 : Income Statement and Statement of Comprehensive Income

53
Q

On December 31 of the current year, the Board of Directors of Maxx Manufacturing, Inc. committed to a plan to discontinue the operations of its Alpha division in the following year. Maxx estimated that Alpha’s following year operating loss would be $500,000 and that Alpha’s facilities would be sold for $300,000 less than their carrying amounts. Alpha’s current-year operating loss was $1,400,000 before any consideration of impairment loss. Maxx’s effective tax rate is 30%. These estimates were accurate.

In its following year income statement, what amount should Maxx report as loss from discontinued operations?

B $500,000
C $560,000
D $800,000

A $350,000

A

A $350,000

A gain or loss on disposal would be recorded in the year of sale. However, if the entity is already aware that the discontinued operations would be sold at lesser than carrying value, there is an impairment loss that would be recognized in the Income Statement in the year the entity becomes aware of the impairment.

As the fair value of Alpha’s facilities was $300,000 less than the carrying value, there has been an impairment loss, and that loss should be recognized in the current year along with the operating loss.

Loss from Discontinued Operations in the following year:

Gain or Loss from Operations
($500,000)
Impairment Loss -
Gain or Loss on Disposal -
Income from Discontinued Operations, Gross of Tax ($500,000)
Add/Less: Tax Effects $150,000
Income from Discontinued Operations, Net of Tax ($350,000)

Note: Loss from Discontinued Operations in the current year:

Gain or Loss from Operations ($1,400,000)
Impairment Loss ($300,000)
Gain or Loss on Disposal -
Income from Discontinued Operations, Gross of Tax ($1,700,000)
Add/Less: Tax Effects $510,000
Income from Discontinued Operations, Net of Tax
($1,190,000)

Question # 108 | Blueprint Area: 1 A iii : Income Statement/Statement of Profit or Loss and Statement of Comprehensive IncomeRelated Chapter: FAR-2 : Income Statement and Statement of Comprehensive Income

54
Q

The following information pertains to Deal Corp.’s current year cost of goods sold:

Inventory, 12/31 of the previous year $ 90,000
Purchases 124,000
Write-off of obsolete inventory 34,000
Inventory, 12/31 of current year 30,000
The inventory written off became obsolete due to an unexpected and unusual technological advance by a competitor. In its year-end income statement, what amount should Deal report as cost of goods sold?

A $218,000
B $184,000

D $124,000

C $150,000

A

C $150,000

When closing inventory is subtracted from the total inventory available, the result would be the Cost of Goods Sold (COGS). Total inventory includes opening inventory and purchases, reduced by the write-off of obsolete inventory.

COGS = Opening inventory + Purchases - Write-off of obsolete inventory - Closing inventory

= $90,000 + $124,000 - $34,000 - $30,000 = $150,000.

Inventory, 12/31 of the previous year $ 90,000
Add: Purchases 124,000
Goods available for sale $214,000
Less: Inventory, 12/31 of current year $30,000
Write-off of obsolete inventory 34,000 (64,000)
Cost of Goods Sold $150,000

Question # 100 | Blueprint Area: 1 A iii : Income Statement/Statement of Profit or Loss and Statement of Comprehensive IncomeRelated Chapter: FAR-2 : Income Statement and Statement of Comprehensive Income

55
Q

On July 1, year 1, Clark Company borrowed 1,680,000 local currency units (LCUs) from a foreign lender, evidenced by an interest bearing note due on July 1, year 2, which is denominated in the currency of the lender. The U.S. dollar equivalent of the note principal was as follows:
Date Amount
7/1, year 1 (date borrowed) $210,000
12/31, year 1 (Clark’s year end) 240,000
7/1, year 2 (date repaid) 280,000In its income statement for year 2, what amount should Clark include as a foreign exchange gain or loss?
A $70,000 gain
B $70,000 loss
C $40,000 gain
D $40,000 loss

A

D $40,000 loss

A transaction gain or loss (measured from the transaction date or most recent balance sheet date, whichever is later) is realized upon settlement of a foreign currency transaction. In year 2, a loss of $40,000 (i.e., $240,000 – $280,000) is recorded due to the additional amount that must be repaid since the last balance sheet date.

Question # 154 | Blueprint Area: 1 A iii : Income Statement/Statement of Profit or Loss and Statement of Comprehensive IncomeRelated Chapter: FAR-11 : Foreign Currency Transactions

56
Q

On March 15, year 2, Krol Co. paid property taxes of $90,000 on its office building for the calendar year 2. On April 1, year 2, Krol paid $150,000 for unanticipated repairs to its office equipment. The repairs will benefit operations for the remainder of year 2. What is the total amount of these expenses that Krol should include in its quarterly income statement for the three months ended June 30, year 2?

A $172,500
B $ 97,500

D $ 37,500

C $ 72,500

A

C $ 72,500

Annual property taxes should be accrued or deferred at each interim reporting date to provide an appropriate cost in each period and allocated ratably to each interim period of the year. Because the unanticipated repairs to the office equipment benefit the last three quarters of the year, each of these periods should be charged with an appropriate portion of the cost by the use of accruals or deferrals.

Annual property taxes ($90,000 / 4) $22,500
Unanticipated repairs incurred in April ($150,000 / 3) 50,000
Expense reported in 6/30, year 2 income statement $72,500

Question # 149 | Blueprint Area: 1 A iii : Income Statement/Statement of Profit or Loss and Statement of Comprehensive IncomeRelated Chapter: FAR-2 : Income Statement and Statement of Comprehensive Income

57
Q

On April 30 of the current year, Carty Corp. approved a plan to dispose of a segment of its business. The estimated disposal loss is $480,000, including severance pay of $55,000 and employee relocation costs of $25,000, both of which are directly associated with the decision to dispose of the segment. Also included is the segment’s estimated operating loss of $100,000 for the period from May 1 to the disposal date November 30. A $120,000 operating loss from January 1 to April 30 is not included in the estimated disposal loss of $480,000.

Before income taxes, what amount should be reported in Carty’s income statement for the current year ended December 31 as the loss from discontinued operations?

A $700,000

C $480,000
D $455,000

B $600,000

A

B $600,000

In the period in which a component of an entity either has been disposed of or is classified as held for sale, the income statement of a business enterprise for current and prior periods shall report the results of operations of the component in discontinued operations in the period(s) in which they occur. Thus, the $120,000 operating loss incurred before April 30 is included in the loss from discontinued operations as well as the $100,000 loss from May 1 to November 30.

The $55,000 of severance pay and the $25,000 of employee relocation expenses are (1) clearly a direct result of the decision to dispose of the segment and (2) are clearly not the adjustments of carrying amounts or costs, or expenses that should have been recognized on a going-concern basis prior to the measurement date. Therefore, they are included in the loss from discontinued operations. Operating loss through April 30 ($120,000) + loss on disposal, including operations from May 1 to November 30 ($480,000) = pretax loss from discontinued operations ($600,000). As the loss on operations from May 1 to November 30 is included in the $480,000 loss on disposal, it should not be added in again.

Question # 106 | Blueprint Area: 1 A iii : Income Statement/Statement of Profit or Loss and Statement of Comprehensive IncomeRelated Chapter: FAR-2 : Income Statement and Statement of Comprehensive Income

58
Q

Gains resulting from the process of translating a foreign entity’s financial statements from the functional currency, which has not experienced significant inflation, to U.S. dollars should be included as a (an)

B Deferred credit
C Component of income from continuing operations
D Extraordinary item
A Other comprehensive income item

A

A Other comprehensive income item
If an entity’s functional currency is the foreign currency, and it has not experienced significant inflation, translation adjustments result from the process of translating that entity’s financial statements into the reporting currency. Translation adjustments (gains or losses) are not included in net income, but in other comprehensive income.

Question # 152 | Blueprint Area: 1 A iii : Income Statement/Statement of Profit or Loss and Statement of Comprehensive IncomeRelated Chapter: FAR-2 : Income Statement and Statement of Comprehensive Income

59
Q

A component that has been sold, abandoned, or spun-off would be reported in which of the following?

A Continuing operations

C Extraordinary items
D None of the above

B Discontinued operations

A

B Discontinued operations
The disposal of a component of the entity, which represents separate operations and cash flows, is reported in discontinued operations. A component that has been sold, abandoned, or spun-off is an example of an item that would be reported in discontinued operations. The component’s operations and cash flows must be clearly distinguished.

Question # 118 | Blueprint Area: 1 A iii : Income Statement/Statement of Profit or Loss and Statement of Comprehensive IncomeRelated Chapter: FAR-2 : Income Statement and Statement of Comprehensive Income

60
Q

Gains that are unusual and infrequent should be reported as a direct increase to which of the following?

B Comprehensive income
C Operating Income
D Income from discontinued operations, net of tax
A Income from continuing operations

A

A Income from continuing operations

As part of its simplification initiative, FASB issued Accounting Standards Update 2015-01 which eliminates the concept of an extraordinary item from U.S. GAAP. These formerly “extraordinary” gains/losses (i.e., gains/losses which are unusual AND infrequent) are now reported as part of Non-Operating gains/losses.

Option (B) is incorrect because comprehensive income includes net income and other comprehensive income.
Option (C) is incorrect as unusual and infrequent gains are reported as a part of Non-Operating gains/losses and not operating income.
Option (D) is incorrect because discontinued operations are reported separately.

Question # 123 | Blueprint Area: 1 A iii : Income Statement/Statement of Profit or Loss and Statement of Comprehensive IncomeRelated Chapter: FAR-2 : Income Statement and Statement of Comprehensive Income

61
Q

Nola has a portfolio of marketable equity securities where she does not exercise significant influence. How should Nola report unrealized gains and losses from these securities?

A Report as a separate component of other comprehensive income.

C Report as a component of income from discontinued operations.
D Do not recognize any unrealized gains or losses.

B Report as a component of income from continuing operations.

A

B Report as a component of income from continuing operations.
If there is no significant influence, Equity Securities are reported using Fair Value Method (FVTNI) with Unrealized Gains and Losses reported in Income Statement.

If there is significant influence, Equity Securities are reported using Equity Method with no unrealized gains and losses reported unless a Fair Value Election is made in which case Unrealized Gains and Losses reported in Income Statement.

Equity Investments are as reported as follows:

Question # 87 | Blueprint Area: 1 A iii : Income Statement/Statement of Profit or Loss and Statement of Comprehensive IncomeRelated Chapter: FAR-2 : Income Statement and Statement of Comprehensive Income

62
Q

Palms Co. purchased raw material from a vendor in Mexico on July 20 for 275,000 pesos. The payment was due in pesos on February 28. The spot rates on different dates are as under:

July 20 $1.25
December 31 $1.32
February 28 $1.17
How should the transaction gain be reported on financial statements at year-end?

A A gain of $19,250 on the statement of income
B A loss of $19,250 as a part of other comprehensive income
C A gain of $19,250 as a part of other comprehensive income
D A loss of $19,250 on the statement of income

A

D A loss of $19,250 on the statement of income

The purchase of raw materials means Palms Co. is liable to pay for it on February 28. On the initial measurement on July 20, it must have recognized an accounts payable of (275,000 x 1.25) = $343,750. However, the forex rate changed on December 31, making the accounts payable increase by $19,250.

Since Palms Co. will have to pay $19,250 on Dec 31, it is a loss. Also, such a loss is always recognized on the income statement and not on the other comprehensive income.

Question # 179 | Blueprint Area: 1 A iii : Income Statement/Statement of Profit or Loss and Statement of Comprehensive IncomeRelated Chapter: FAR-11 : Foreign Currency Transactions

63
Q

In September of the current year, Koff Co.’s operating plant was destroyed by an earthquake. The portion of the resultant loss not covered by insurance was $700,000. Koff’s income tax rate for the year is 21%. In its year-end income statement, what should Koff report as a loss?

A $0
B $280,000
C $700,000
D $420,000

A

C $700,000

The loss is $700,000. The tax rate is not considered in the calculation and is included as a distractor, along with the fact that it’s an earthquake.

Question # 122 | Blueprint Area: 1 A iii : Income Statement/Statement of Profit or Loss and Statement of Comprehensive IncomeRelated Chapter: FAR-2 : Income Statement and Statement of Comprehensive Income

64
Q

In Baer Food Co.’s current year single-step income statement, the section titled “Revenues” consisted of the following:

Net sales revenue $187,000
Results from discontinued operations:
Loss from operations of segment (net of $1,200 tax effect) $(2,400)
Gain on disposal of segment (net of $7,200 tax effect) 14,400 12,000
Interest revenue 10,200
Gain on sale of equipment 4,700
Cumulative change in Yr8 and Yr9 income due to change in 1,500
depreciation method (net of $750 tax effect)
Total revenues $215,400
In the revenue section of the income statement, Baer Food should have reported total revenues of

A $216,300
B $215,400
C $203,700
D $201,900

A

D $201,900

Question # 126 | Blueprint Area: 1 A iii : Income Statement/Statement of Profit or Loss and Statement of Comprehensive IncomeRelated Chapter: FAR-2 : Income Statement and Statement of Comprehensive Income

65
Q

The following information pertains to Flint Co.’s sale of 10,000 foreign currency units under a forward contract dated November 1, of the current year for delivery on January 31 of the following year:
11/1 12/31
Spot-rate $0.80 $0.83
30-day future rates 0.79 0.82
90-day future rates 0.78 0.81Flint entered into the forward contract in order to speculate in the foreign currency. In Flint’s income statement for the current year ended December 31, what amount of loss should be reported from this forward contract?

B $300
C $200
D $0

A $400

A

A $400

A forward exchange contract (forward contract) is an agreement to exchange different currencies at a specified future date and at a specified rate (the forward rate). A forward contract is a foreign currency transaction. Therefore, a gain or loss on a forward contract is included in determining net income in accordance with the requirements for other foreign currency transactions. A gain or loss on a speculative forward contract (that is, a contract that does not hedge an exposure) is computed by multiplying the foreign currency amount of the contract by the difference between the forward rate available for the remaining maturity of the contract and the contracted forward rate (or the forward rate last used to measure a gain or loss on that contract for an earlier period).
Foreign currency units 10,000
Times: excess of forward rate available for the remaining maturity of the contract and the contracted forward rate ($0.82 - $0.78) × $0.04
Loss on forward contract $ 400

Question # 156 | Blueprint Area: 1 A iii : Income Statement/Statement of Profit or Loss and Statement of Comprehensive IncomeRelated Chapter: FAR-11 : Foreign Currency Transactions

66
Q

On December 31 of the current year, the Board of Directors of Maxx Manufacturing, Inc. committed to a plan to discontinue the operations of its Alpha division in the following year. Maxx estimated that Alpha’s following year operating loss would be $500,000 and that Alpha’s facilities would be sold for $300,000 less than their carrying amounts. Alpha’s current year operating loss was $1,400,000 before any consideration of impairment loss. Maxx’s effective tax rate is 30%. These estimates were accurate.

In its current year income statement, what amount should Maxx report as loss from discontinued operations?

B $1,400,000
C $1,540,000
D $2,200,000
A $1,190,000

A

A $1,190,000

The income statement of a business enterprise for current and prior periods shall report the results of operations of the component in discontinued operations in the period(s) in which they occur.

Alpha’s operating loss (before consideration of impairment) $1,400,000
Impairment loss (recognized in current year) 300,000
Loss on discontinued operation, before taxes $1,700,000
Applicable income tax benefit ($1,700,000 × 0.30) (510,000)
Loss on discontinued operation, net of taxes $1,190,000

Question # 107 | Blueprint Area: 1 A iii : Income Statement/Statement of Profit or Loss and Statement of Comprehensive IncomeRelated Chapter: FAR-2 : Income Statement and Statement of Comprehensive Income

67
Q

On September 1, Brady Corp. entered into a foreign exchange contract for speculative purposes by purchasing 50,000 deutsche marks for delivery in 60 days. The rates to exchange $1 for 1 deutsche mark follow:
9/1 9/30
Spot rate $0.75 $0.70
30-day forward rate $0.73 $0.72
60-day forward rate $0.74 $0.73In its September 30 income statement, what amount should Brady report as foreign exchange loss?
A $2,500
B $1,500

D $ 500

C $1,000

A

C $1,000

A gain or loss on a speculative forward exchange contract is included in determining income from continuing operations in accordance with the requirements for other foreign currency transactions. A gain or loss on a speculative forward contract is computed by multiplying the foreign currency amount of the contract by the difference between the forward rate available for the remaining maturity of the contract and the contracted forward rate (or the forward rate last used to measure a gain or loss on that contract for an earlier period). Thus, the amount that Brady should report as foreign exchange loss for the month ended 9/30 is determined as follows:
Foreign currency units purchased under speculative
forward contract $50,000
Times excess of contracted forward rate over forward rate
available for remaining portion of contract ($0.74 - $0.72) × 0.02
Foreign exchange loss recognized September $ 1,000

Question # 157 | Blueprint Area: 1 A iii : Income Statement/Statement of Profit or Loss and Statement of Comprehensive IncomeRelated Chapter: FAR-11 : Foreign Currency Transactions

68
Q

Midway Corp. had the following transactions during year 2:

$800,000 pretax loss on equipment damaged by a hurricane. Midway also received $1,000,000 from its insurance company to replace a building, with a carrying value of $300,000, that had been destroyed by the hurricane.
What amount should Midway report in its year 2 income statement as an ordinary loss before income taxes?

B $1,300,000
C $1,800,000
D $2,500,000
A $ 100,000

A

A $ 100,000

The pretax loss on the equipment damaged in the hurricane must be offset by the excess of the building insurance proceeds over the carrying amount.

Total ordinary loss: $100,000

Pretax loss on damaged equipment $800,000
Proceeds from insurance company $1,000,000
Carrying amount of building (300,000)
Less: pretax gain on building destroyed by hurricane (700,000)
Ordinary loss before income taxes $100,000

Question # 119 | Blueprint Area: 1 A iii : Income Statement/Statement of Profit or Loss and Statement of Comprehensive IncomeRelated Chapter: FAR-2 : Income Statement and Statement of Comprehensive Income

69
Q

Ball Corp. had the following foreign currency transactions during the current year:

Merchandise was purchased from a foreign supplier on January 20 for the U.S. dollar equivalent of $90,000. The invoice was paid on March 20 at the U.S. dollar equivalent of $96,000.
On July 1, Ball borrowed the U.S. dollar equivalent of $500,000 evidenced by a note that was payable in the lender’s local currency on July 1 in two years. On December 31, the U.S. dollar equivalents of the principal amount and accrued interest were $520,000 and $26,000, respectively. Interest on the note is 10% per annum.
In Ball’s year-end income statement, what amount should be included as foreign exchange loss?

A $0
B $ 6,000
C $21,000
D $27,000

A

D $27,000

All Foreign Exchange Transaction Losses whether realized or unrealized would be recorded in Income Statement. Foreign Exchange Translation (not transaction) Gains and Losses are parked in Other Comprehensive Income.

The payables resulting from the merchandise purchased from the foreign supplier and the borrowing are transactions denominated in a foreign currency. The payable from the merchandise purchased was recorded at $90,000 on 1/20. It was paid on 3/20 at the U.S. dollar equivalent of $96,000, resulting in a $6,000 foreign exchange loss.

The payable from the borrowing was recorded at $500,000 at 7/1. Accrued interest on the borrowing was $25,000 ($500,000 × 10% × 6/12) at 12/31. At 12/31, the U.S. dollar equivalents of principal and accrued interest were $520,000 and $26,000, respectively, resulting in an additional $21,000 [($520,000 + $26,000) - ($500,000 + $25,000)] foreign exchange loss.

Thus, the total amount of foreign exchange loss to be recognized in the year end income statement is $27,000 ($6,000 + $21,000).

Question # 153 | Blueprint Area: 1 A iii : Income Statement/Statement of Profit or Loss and Statement of Comprehensive IncomeRelated Chapter: FAR-11 : Foreign Currency Transactions

70
Q

Data regarding Ball Corp.’s available-for-sale debt securities is as follows:

Year Cost Market Value
December 31, Year 5 $150,000 $130,000
December 31, Year 6 $150,000 $160,000
Differences between cost and market values are considered temporary. Ball does not elect the fair value option to account for available-for-sale securities. Ball’s Year 6 other comprehensive income would be: ?

B $20,000
C $10,000
D $0

A $30,000

A

A $30,000

Investments in debt securities that are classified as available for sale (AFS) securities are valued at the fair market value (FMV) on the balance sheet date. If fair value option is not elected, their unrealized gain or loss is reported in the statement of comprehensive income. At the end of Year 5, Ball Corp. would have valued AFS securities at $130,000, which is FMV on that date. At the end of Year 6, they would be valued at $160,000. The resultant gain of $30,000 would be reported in the statement of comprehensive income for Year 6.
Option (B) is incorrect because $20,000 would be the difference between the cost $150,000 and market value at the end of Year 5 $130,000. Any unrealized gain or loss of Year 5 would be reported in the statement of comprehensive income of that year and will not affect the statement of comprehensive income for Year 6.
Option (C) is incorrect because $10,000 is the difference between the cost $150,000 and the FMV at the end of Year 6 $160,000. Investments that are classified as AFS would be valued at the FMV as of each balance sheet date.
Option (D) is incorrect because $0 implies that no unrealized gain or loss is recognized in the statement of comprehensive income in Year 6. Ball Corp. had not elected the fair value option, any unrealized gain or loss would be reported in the statement of comprehensive income and not the statement of income.

Question # 88 | Blueprint Area: 1 A iii : Income Statement/Statement of Profit or Loss and Statement of Comprehensive IncomeRelated Chapter: FAR-2 : Income Statement and Statement of Comprehensive Income

71
Q

Which of the following statements regarding foreign exchange gains and losses is correct?
A An exchange gain occurs when the exchange rate increases between the date a payable is recorded and the date of cash payment.

C An exchange loss occurs when the exchange rate decreases between the date a payable is recorded and the date of the cash payment.
D An exchange loss occurs when the exchange rate increases between the date a receivable is recorded and the date of the cash receipt.

B An exchange gain occurs when the exchange rate increases between the date a receivable is recorded and the date of cash receipt.

A

B An exchange gain occurs when the exchange rate increases between the date a receivable is recorded and the date of cash receipt.

Receivable: Rate increase results in a gain (receive more at settlement), rate decrease results in a loss (receive less at settlement). Payable: Rate increase results in a loss (pay more at settlement), rate decrease results in a gain (pay less at settlement).

Question # 163 | Blueprint Area: 1 A iii : Income Statement/Statement of Profit or Loss and Statement of Comprehensive IncomeRelated Chapter: FAR-11 : Foreign Currency Transactions

72
Q

Shore Co. records its transactions in U.S. dollars. A sale of goods resulted in a receivable denominated in Japanese yen, and a purchase of goods resulted in a payable denominated in euros. Shore recorded a foreign exchange gain on collection of the receivable and an exchange loss on settlement of the payable. The exchange rates are expressed as so many units of foreign currency to one dollar. Did the number of foreign currency units exchangeable for a dollar increase or decrease between the contract and settlement dates?
# Yen exchangable for $1 Euros exchangable for $1
A Increase Increase

C Decrease Increase
D Increase Decrease

B Decrease Decrease

A

B Decrease Decrease

To record a foreign exchange gain on collection of the receivable, the exchange rate of Japanese yen to one U.S. dollar must have decreased. For instance, assume a receivable for l,000 yen and an exchange rate of 10 yen to 1 dollar at the contract date. If the exchange rate changed to 5 yen to 1 dollar, the 1,000 yen when collected could be converted into 200 U.S. dollars at the settlement date. This $200 is twice the $100 that could have been obtained from the conversion of currency at the contract date (1,000 / 10 = $100). To record an exchange loss on settlement of the payable, the exchange rate of euros to one dollar must have decreased. For example, assume a payable for 1,000 euros and an exchange rate of 4 euros to 1 dollar at the contract date. If the exchange rate changed to 2 euros to 1 dollar, the 1,000 euross paid at the settlement date required more U.S. dollars than would have been required at the date of contract (1,000 / 2 = $500 at date of settlement versus 1,000 / 4 = $250 at date of contract).

Question # 155 | Blueprint Area: 1 A iii : Income Statement/Statement of Profit or Loss and Statement of Comprehensive IncomeRelated Chapter: FAR-11 : Foreign Currency Transactions

73
Q

Dodd Co.’s debt securities at December 31 included available-for-sale securities with a cost basis of $24,000 and a fair value of $30,000. Dodd’s income tax rate was 20%. What amount of unrealized gain or loss should Dodd recognize in its income statement at December 31?

A $6,000 loss.

C $4,800 gain.
D $6,000 gain.
B $0

A

B $0

Unrealized gains and losses on certain investments in debt securities including the following are reported exclusively as other comprehensive income:

Unrealized holding gains and losses on available-for-sale (AFS) debt securities

Unrealized holding gains and losses that result from a debt security being transferred into the AFS category from the HTM category
Subsequent decreases or increases in the fair value of AFS debt securities previously written down as impaired
A change in the market value of a futures contract that qualifies as a hedge of an asset reported at fair value
Such gains and losses are not classified as net income unless the securities are sold off
Going by the above principle, the unrealized gain on the debt securities held by Dodd Co. shouldn’t be recognized in its income statement on December 31, but will be classified in other comprehensive income.

Question # 176 | Blueprint Area: 1 A iii : Income Statement/Statement of Profit or Loss and Statement of Comprehensive IncomeRelated Chapter: FAR-2 : Income Statement and Statement of Comprehensive Income

74
Q

A transaction that is unusual in nature or infrequent in occurrence should be reported as a(an)

A Component of income from continuing operations, net of applicable income taxes.
B Extraordinary item, net of applicable income taxes.

D Extraordinary item, but not net of applicable income taxes.

C Component of income from continuing operations, but not net of applicable income taxes.

A

C Component of income from continuing operations, but not net of applicable income taxes.

A transaction that is unusual in nature or infrequent in occurrence should be reported as a component of income from continuing operations. It cannot be shown as net of taxes as the tax is a separate line item to arrive at the income from continuing operations.

Option (A) is incorrect because components of income from continuing operations are not shown net of taxes.
Option (B) and (D) are incorrect because unusual and infrequent gains or losses were previously reported as extraordinary items, which has been eliminated from GAAP.

Question # 105 | Blueprint Area: 1 A iii : Income Statement/Statement of Profit or Loss and Statement of Comprehensive IncomeRelated Chapter: FAR-2 : Income Statement and Statement of Comprehensive Income

75
Q

Zach Corp. pays commissions to its sales staff at the rate of 3% of net sales. Sales staff are not paid sala­ries but are given monthly advances of $15,000. Advances are charged to commission expense, and recon­ciliations against commissions are prepared quarterly. Net sales for the current year ended March 31 were $15,000,000. The unadjusted balance in the commissions expense account on March 31 was $400,000. March advances were paid on April 3. In its income statement for the current year ended March 31, what amount should Zach report as commission expense?

A $465,000

C $415,000
D $400,000

B $450,000

A

B $450,000

Accrual accounting recognizes expenses in the period they are incurred rather than when paid. Zach pays commissions to its sales staff at the rate of 3% of net sales. For the year ended 3/31, Zach should report commission expense of $450,000 ($15,000,000 × 3%).

Question # 96 | Blueprint Area: 1 A iii : Income Statement/Statement of Profit or Loss and Statement of Comprehensive IncomeRelated Chapter: FAR-2 : Income Statement and Statement of Comprehensive Income

76
Q

During a reporting period, a computer manufacturing company used raw materials of $50,000, had direct labor costs of $75,000, and factory overhead of $30,000. Other expenses were for advertising of $5,000, staff salaries of $10,000 and bad debt of $3,000. The company did not have a beginning balance in any inventory account. All goods manufactured during the period were sold during the period. What amount was the company’s cost of goods sold during the reporting period?

B $160,000
C $170,000
D $173,000

A $155,000

A

A $155,000

The company did not have a beginning balance in any inventory account. All goods manufactured during the period were sold during the period. This implies that both the opening and the ending inventory were zero. Goods were not purchased but were manufactured. Cost of goods manufactured includes the raw material cost of $50,000, labor cost of $75,000 and factory overhead of $30,000.

Cost of goods sold (COGS) = Beginning inventory + Cost of goods purchased + Direct manufacturing cost – Ending inventory.
Total cost of goods sold = $0 + $50,000 + $75,000 + 30,000 - $0 = $155,000.
Advertising expenses of $5,000 and bad debt of $3,000 are selling expenses, whereas staff salaries of $10,000 are general and administrative expenses. All these form a part of operating expenses and are not to be taken into consideration for calculating the COGS.

Question # 173 | Blueprint Area: 1 A iii : Income Statement/Statement of Profit or Loss and Statement of Comprehensive IncomeRelated Chapter: FAR-2 : Income Statement and Statement of Comprehensive Income

77
Q

A company has the following items on its year-end trial balance:

Net sales $500,000
Common stock 100,000
Insurance expense 75,000
Wages 50,000
Cost of goods sold 100,000
Cash 40,000
Accounts payable 25,000
Interest payable 20,000
What is the company’s gross profit?

A $230,000
B $275,000

D $500,000

C $400,000

A

C $400,000

Gross Profit = Net sales - COGS ($500,000 - $100,000) = $400,000.
Option (A) is incorrect because insurance expense, wages, accounts payable and interest payable have been included.
Option (B) is incorrect because insurance expense, wages have been included.
Option (D) is incorrect because it does not subtract the Cost of Goods Sold (COGS).

Question # 81 | Blueprint Area: 1 A iii : Income Statement/Statement of Profit or Loss and Statement of Comprehensive IncomeRelated Chapter: FAR-2 : Income Statement and Statement of Comprehensive Income

78
Q

Which of the following would be reported in the income statement of a proprietorship?
# Proprietor’s draw Depreciation
A Yes Yes
B Yes No

D No No
C No Yes

A

C No Yes

A sole proprietorship’s equity consists of a single proprietor’s equity account, Owner’s Equity or Net Worth. This is the sum of the beginning capital balance, plus additional investments during the period, plus net income (or minus net loss) minus withdrawals. The proprietor’s draw is not reported separately on the income statement, but rather is included in Owner’s Equity or Net Worth. Depreciation is not included in the Owner’s Equity or Net Worth account. It is reported on the income statement of a proprietorship.

Question # 95 | Blueprint Area: 1 A iii : Income Statement/Statement of Profit or Loss and Statement of Comprehensive IncomeRelated Chapter: FAR-2 : Income Statement and Statement of Comprehensive Income

79
Q

On February 2, Flint Corp.’s board of directors voted to discontinue operations of its frozen food division and to sell the division’s assets on the open market as soon as possible. The division reported net operating losses of $20,000 in January and $30,000 in February. On February 26, sale of the division’s assets resulted in a gain of $90,000. What amount of gain or loss from discontinued operations should Flint recognize in its income statement for the three months ended March 31?

A $0

C $60,000
D $90,000

B $40,000

A

B $40,000

The Gain or Loss from Discontinued Operations would be calculated as follows:

Gain from sale of division assets on Feb 26 $90,000
Less: Operating loss during the fiscal year (50,000)
Gain or Loss from Discontinued Operations $40,000

Question # 114 | Blueprint Area: 1 A iii : Income Statement/Statement of Profit or Loss and Statement of Comprehensive IncomeRelated Chapter: FAR-2 : Income Statement and Statement of Comprehensive Income

80
Q

Which of the following is a component of other comprehensive income?

A Minimum accrual of vacation pay.

C Changes in market value of inventory.
D Unrealized gain or loss on trading debt securities.

B Cumulative currency translation adjustments.

A

B Cumulative currency translation adjustments.

Items (revenues, expenses, gains, and losses) that previously were included in the equity section as a separate component of owners’ equity are required to be reported in other comprehensive income. Examples include foreign currency items, pension adjustments, unrealized gains and losses on certain investments in debt and equity securities, and gains and losses on cash flow hedging derivative instruments.

Question # 140 | Blueprint Area: 1 A iii : Income Statement/Statement of Profit or Loss and Statement of Comprehensive IncomeRelated Chapter: FAR-2 : Income Statement and Statement of Comprehensive Income

81
Q

Which of the following assets or transactions is an element of comprehensive income?

A Investments by owners

C Distributions to owners
D Deferred revenue

B Sales revenue

A

B Sales revenue
Comprehensive income includes all changes in equity during a period except those resulting from investments by owners and distributions to owners. Sales revenue would change equity during the period where­as deferred revenue would not.

Options (A), (C) and (D) are incorrect based on the above explanation.

Question # 137 | Blueprint Area: 1 A iii : Income Statement/Statement of Profit or Loss and Statement of Comprehensive IncomeRelated Chapter: FAR-2 : Income Statement and Statement of Comprehensive Income

82
Q

On January 1, year 6, Mill Co. exchanged equipment for a $200,000 non-interest-bearing note due on January 1, year 9. The prevailing rate of interest for a note of this type at January 1, year 6 was 10%. The present value of $1 at 10% for three periods is 0.75. What amount of interest revenue should be included in Mill’s year 7 income statement?

A $0
B $15,000

D $20,000

C $16,500

A

C $16,500

Question # 85 | Blueprint Area: 1 A iii : Income Statement/Statement of Profit or Loss and Statement of Comprehensive IncomeRelated Chapter: FAR-2 : Income Statement and Statement of Comprehensive Income

83
Q

Hunt Co. purchased merchandise for £300,000 from a vendor in London on November 30 of the current year. Payment in British pounds was due on January 30 of the next year. The exchange rates to purchase one pound were as follows:
Nov. 30 Dec. 31
Spot-rate $1.65 $1.62
30-day rate 1.64 1.59
60-day rate 1.63 1.56In its December 31, current year income statement, what amount should Hunt report as foreign exchange gain?
A $12,000

C $6,000
D $0

B $9,000

A

B $9,000

Whenever a transaction is denominated (i.e., payable) in a foreign currency, changes in the translation rate (i.e., the spot rate) of the foreign currency with respect to the entity’s functional currency (i.e., the dollar in this case) will result in a gain or loss. The gain or loss should be recognized in income in the period(s) the rate changes, and the related asset or liability (i.e., accounts payable in this case) should be adjusted accordingly.
Initial obligation, 11/30, in U.S. dollars

(£300,000 × $1.65) $ 495,000
Amount payable, 12/31, in U.S. dollars

(£300,000 × $1.62) (486,000)
Foreign exchange gain recognized $ 9,000

Question # 159 | Blueprint Area: 1 A iii : Income Statement/Statement of Profit or Loss and Statement of Comprehensive IncomeRelated Chapter: FAR-11 : Foreign Currency Transactions

84
Q

Loss on discontinued operations Investment by owners

According to the FASB conceptual framework, comprehensive income includes which of the following?

A Yes Yes

C No Yes
D No No

B Yes No

A

B Yes No

Comprehensive income includes all changes in equity during a period except those resulting from investments by owners and distributions to owners. Loss on discontinued operations is part of comprehensive income.

Question # 82 | Blueprint Area: 1 A iii : Income Statement/Statement of Profit or Loss and Statement of Comprehensive IncomeRelated Chapter: FAR-2 : Income Statement and Statement of Comprehensive Income

85
Q

Rock Co.’s financial statements had the following balances at December 31:

Ordinary gain $ 50,000
Foreign currency translation gain 100,000
Net Income 400,000
Unrealized gain on available-for-sale debt securities 20,000
What amount should Rock report as comprehensive income for the year ended December 31?

A $400,000
B $420,000

D $570,000

C $520,000

A

C $520,000

Comprehensive income includes all changes in equity during a period except those resulting from investments by owners and distributions to owners. Comprehensive income is divided into net income and other comprehensive income. An entity must classify items of other comprehensive income by their nature, in one of these classifications: foreign currency items, minimum pension liability adjustments, unrealized gains and losses on certain investments in debt securities, and gains and losses on cash flow hedging derivative instruments. All regular items of income and expense, including unusual and infrequently occurring items, are included in the determination of net income. The ordinary gain is already included in the net income amount that is given in this problem. Translation adjustments and unrealized holding gains and losses are reported in other comprehensive income (OCI). They are excluded from current earnings and should not be included in the determination of net income.

Statement of Comprehensive Income
Net Income $400,000
Pension Adjustments $ -
Unrealized Gain or Loss on Available for Sale Securities $20,000
Foreign Currency Translation Gain or Loss $100,000
Effective & Ineffective Portion of Cashflow Hedge $ -
Comprehensive Income $520,000

Question # 135 | Blueprint Area: 1 A iii : Income Statement/Statement of Profit or Loss and Statement of Comprehensive IncomeRelated Chapter: FAR-2 : Income Statement and Statement of Comprehensive Income

86
Q

Which of the following statements concerning other comprehensive income (OCI) is false?

A The recognition and measurement of other comprehensive income is based upon GAAP.

C The effective portion of gains and losses on derivatives designated as and qualifying as cash flow hedges is reported in other comprehensive income.
D An entity must display the accumulated balance of other comprehensive income separately from retained earnings, capital stock, and additional paid-in capital in the equity section of a statement of financial position.

B Other comprehensive income has an effect on direct adjustments to equity accounts, such as transactions related to retained earnings.

A

B Other comprehensive income has an effect on direct adjustments to equity accounts, such as transactions related to retained earnings.

Other comprehensive income has no effect on direct adjustments to equity accounts, such as capital stock transactions and transactions related to retained earnings.

Question # 141 | Blueprint Area: 1 A iii : Income Statement/Statement of Profit or Loss and Statement of Comprehensive IncomeRelated Chapter: FAR-2 : Income Statement and Statement of Comprehensive Income

87
Q

On November 1, year 2, Smith Co. contracted to dispose of an industry segment on February 28, year 3. Throughout year 2 the segment had operating losses. These losses were expected to continue until the segment’s disposition. If a loss is anticipated on final disposition, how much of the operating losses should be included in the loss on disposal reported in Smith’s year 2 income statements?

Operating losses for the period January 1 to October 31, year 2.
Operating losses for the period November 1 to December 31, year 2.
Estimated operating losses for the period January 1 to February 28, year 3.
A II only.
B II and III only.
C I and III only.
D I and II only.

A

D I and II only.

In the period in which a component of an entity either has been disposed of or is classified as held for sale, the income statement of a business enterprise for current and prior periods shall report the results of operations of the component in discontinued operations in the period(s) in which they occur. Estimated future operating losses are included in the income statement in the future, when they are incurred.

Question # 151 | Blueprint Area: 1 A iii : Income Statement/Statement of Profit or Loss and Statement of Comprehensive IncomeRelated Chapter: FAR-2 : Income Statement and Statement of Comprehensive Income

88
Q

s

Vane Co.’s trial balance of income statement accounts for the year ended December 31, included the following:

Debit Credit
Sales $575,000
Cost of sales $240,000
Administrative expenses 70,000
Loss on sale of equipment 10,000
Sales commissions 50,000
Interest revenue 25,000
Freight out 15,000
Loss on early retirement of long-term debt 20,000
Uncollectible accounts expense 15,000
Totals $420,000 $600,000
Other information:
Finished goods inventory:
January 1 $400,000
December 31 360,000
Vane’s income tax rate is 21%. In Vane’s year-end multiple-step income statement, what amount should Vane report as income after income taxes from continuing operations?

B $129,500
C $140,000
D $147,000

A $142,200

A

A $142,200

89
Q

Palmyra Co. has net income of $11,000, a positive $1,000 net cumulative effect of a change in accounting principle, a $3,000 unrealized loss on available-for-sale securities, a positive $2,000 foreign currency translation adjustment, and a $6,000 increase in its common stock. What amount is Palmyra’s comprehensive income?

A $4,000

C $11,000
D $17,000

B $10,000

A

B $10,000

Comprehensive income includes all changes in equity during a period except those resulting from investments by owners and distributions to owners.
Comprehensive income is divided into net income and other comprehensive income (OCI). OCI includes revenues, expenses, gains, and losses that are excluded from net income. Instead, they are listed after net income on the income statement. These items have not yet been realized. In other words, the underlying transaction has not been completed or settled yet.

Examples include: foreign currency items; certain pension adjustments; unrealized gains and losses on certain investments; and gains and losses on derivatives. Comprehensive income is $10,000 ($11,000 – 3,000 + 2,000). The cumulative effect of a change in accounting principle would be applied to beginning retained earnings. Increases in common stock are not reflected in income, but rather on the balance sheet.

Question # 143 | Blueprint Area: 1 A iii : Income Statement/Statement of Profit or Loss and Statement of Comprehensive IncomeRelated Chapter: FAR-2 : Income Statement and Statement of Comprehensive Income

90
Q

A company’s activities for year 2 included the following:

Gross sales $3,600,000
Cost of goods sold 1,200,000
Selling and administrative expense 500,000
Adjustment for a prior-year understatement of amortization expense 59,000
Sales returns 34,000
Gain on sale of available-for-sale debt securities 8,000
Gain on disposal of a discontinued business segment 4,000
Unrealized gain on available-for-sale debt securities 2,000
The company has a 30% effective income tax rate. What is the company’s net income for year2?

A $1,267,700
B $1,273,300

D $1,316,000

C $1,314,600

A

C $1,314,600

Question # 132 | Blueprint Area: 1 A iii : Income Statement/Statement of Profit or Loss and Statement of Comprehensive IncomeRelated Chapter: FAR-2 : Income Statement and Statement of Comprehensive Income

91
Q

For the eight months ended August 31, year 5, the carpet division of a flooring company, which is considered a major line of business, had an operating loss of $115,000 from operations. On September 1, year 5, the board of directors voted to discontinue the division’s operations. On December 31, year 5, the division was sold for a pre-tax loss of $135,000. The division’s operating loss for year 5 was $240,000. The company’s income tax rate is 30%. What amount of loss should the company report as discontinued operations in the December 31, year 5, income statement?

B $260,000
C $182,000
D $168,000

A $262,500

A

A $262,500

Results of all discontinued operations are reported net of tax as a separate component on the income statement. The loss to be reported as discontinued operations by the flooring company in question is as follows:

Loss on sale of division $135,000
Add: Division’s operating loss $240,000
Total loss from discontinued operations $375,000
Less: Income Tax @ 30% ($112,500)
After-tax loss $262,500

Question # 178 | Blueprint Area: 1 A iii : Income Statement/Statement of Profit or Loss and Statement of Comprehensive IncomeRelated Chapter: FAR-2 : Income Statement and Statement of Comprehensive Income

92
Q

One of the elements of a financial statement is comprehensive income. Comprehensive income excludes changes in equity resulting from which of the following?

A Loss from discontinued operations
B Net Income

D Unrealized Gain on Fair Value through Net Income securities

C Dividends paid to stockholders

A

C Dividends paid to stockholders

Comprehensive income includes all items of the income statement and changes in equity during a period except those resulting from investments by owners and distributions to owners.

Question # 134 | Blueprint Area: 1 A iii : Income Statement/Statement of Profit or Loss and Statement of Comprehensive IncomeRelated Chapter: FAR-2 : Income Statement and Statement of Comprehensive Income

93
Q

Which of the following items is included in accumulated other comprehensive income or loss?

A Unrealized gains and losses from the ineffective portion of a derivative properly designated as a fair value hedge
B Unrealized holding gains or losses on debt securities classified as trading securities
C A reduction of shareholders’ equity related to employee stock ownership plans
D Prior service costs not previously recognized as a component of net periodic pension costs

A

D Prior service costs not previously recognized as a component of net periodic pension costs

Prior service costs not previously recognized as a component of net periodic pension costs are included in Accumulated Other Comprehensive Income/Loss. Accumulated Other Comprehensive Income (AOCI) is impacted by unexpected gains and losses from plan assets and unamortized portions of prior service costs from pension plan amendments.

Question # 172 | Blueprint Area: 1 A iii : Income Statement/Statement of Profit or Loss and Statement of Comprehensive IncomeRelated Chapter: FAR-2 : Income Statement and Statement of Comprehensive Income

94
Q

Weald Co. took advantage of market conditions to refund debt. This was the fifth refunding operation carried out by Weald within the last four years. The excess of the carrying amount of the old debt over the amount paid to extinguish it should be reported as a(an)

A Deferred credit to be amortized over life of new debt

C Discontinued Operations, net of income taxes
D Other Comprehensive Income.

B Part of continuing operations

A

B Part of continuing operations

Since the carrying amount of the old debt is greater than the amount paid to extinguish the old debt, the transaction results in a gain. The gain is part of continuing operations.

Question # 93 | Blueprint Area: 1 A iii : Income Statement/Statement of Profit or Loss and Statement of Comprehensive IncomeRelated Chapter: FAR-2 : Income Statement and Statement of Comprehensive Income

95
Q

Which of the following transactions qualify as a discontinued operation?

A Disposal of part of a line of business.

C Phasing out of a production line.
D Changes related to technological improvements.

B Planned and approved sale of a segment.

A

B Planned and approved sale of a segment.

Discontinued operations refers to the operations of a component of an entity that has been disposed of or is still operating, but is the subject of a formal plan for disposal. A component of an entity is a segment, reporting unit, or asset group (not a part of a line of business) whose operations and cash flows are clearly dis­tinguished from the rest of the entity, operationally as well as for financial reporting purposes.

Question # 116 | Blueprint Area: 1 A iii : Income Statement/Statement of Profit or Loss and Statement of Comprehensive IncomeRelated Chapter: FAR-2 : Income Statement and Statement of Comprehensive Income

96
Q

Interest Advertising

Which of the following should be included in General and Administrative expenses?

A Yes No
B Yes Yes
C No Yes
D No No

A

D No No

Interest Expense is classified as a separate line item on the income statement. Advertising is classified as a selling expense.

Question # 168 | Blueprint Area: 1 A iii : Income Statement/Statement of Profit or Loss and Statement of Comprehensive IncomeRelated Chapter: FAR-2 : Income Statement and Statement of Comprehensive Income

97
Q

A company decided to sell an unprofitable division of its business. The company can sell the entire operation for $800,000, and the buyer will assume all assets and liabilities of the operations. The tax rate is 30%. The assets and liabilities of the discontinued operation are as follows:
What is the after-tax net loss on the disposal of the division?

B $ 200,000
C $1,540,000
D $2,200
A $ 140,000

A

A $ 140,000

Question # 117 | Blueprint Area: 1 A iii : Income Statement/Statement of Profit or Loss and Statement of Comprehensive IncomeRelated Chapter: FAR-2 : Income Statement and Statement of Comprehensive Income

98
Q
A

Question # 92 | Blueprint Area: 1 A iii : Income Statement/Statement of Profit or Loss and Statement of Comprehensive IncomeRelated Chapter: FAR-2 : Income Statement and Statement of Comprehensive Income

99
Q

When an entity insures the lives of its key executives, the entity accounts for the life insurance policies in which of the following ways?

A The savings portion of the policies is represented by the growing cash surrender value of the policies, as the entity continues to pay premiums on the policies.
B The entity classifies the cash surrender value of these life insurance policies as noncurrent assets.
C The entity computes its annual life insurance expense by deducting the increase in cash surrender value and deducting any dividends received from the annual premiums paid.
D All of the above

A

D All of the above

The savings portion of key executives’ life insurance policies is represented by the growing cash surrender value of the policies, as the entity continues to pay policy premiums; the entity classifies the cash surrender value of these life insurance policies as noncurrent assets; and the entity computes its annual life insurance expense by deducting the increase in cash surrender value and any dividends received from the annual premiums paid.

Since the answers for A., B., and C. are all correct, the best answer for this question is D., all of the above.

Question # 92 | Blueprint Area: 1 A iii : Income Statement/Statement of Profit or Loss and Statement of Comprehensive IncomeRelated Chapter: FAR-2 : Income Statement and Statement of Comprehensive Income

100
Q

Which of the following items should be shown as a component of comprehensive income?

A Dividend paid to a shareholder.

C Additional capital contribution.
D Deferred revenue

B Foreign currency translation adjustment.

A

B Foreign currency translation adjustment.
Comprehensive Income = Net income + Other comprehensive income.

Other comprehensive income includes foreign currency translation adjustments. Foreign currency translation adjustments for a foreign operation that is relatively self-contained and integrated within its environment do not affect cash flows of the reporting entity. They should be excluded from earnings. Accordingly, translation adjustments are reported in other comprehensive income (OCI). The other items are part of earnings on the income statement

(A) is incorrect because dividend paid to shareholders is a part of shareholders’ equity.

(C) is incorrect because the additional capital contribution is also a part of shareholders’ equity.

(D) is incorrect because deferred revenue is a liability.

Question # 169 | Blueprint Area: 1 A iii : Income Statement/Statement of Profit or Loss and Statement of Comprehensive IncomeRelated Chapter: FAR-2 : Income Statement and Statement of Comprehensive Income

101
Q

During the current year, Fuqua Steel Co. had the following unusual financial events occur:

Bonds payable were retired five years before their scheduled maturity, resulting in a $260,000 gain. Fuqua has frequently retired bonds early when interest rates declined significantly.
A steel forming segment suffered $255,000 in losses due to hurricane damage. This was the fourth similar loss sustained in a 5-year period at that location.
A segment of Fuqua’s operations, steel transportation, was sold at a net loss of $350,000. This was Fuqua’s first divestiture of one of its operating segments.
Before income taxes, what amount of gain (loss) should be reported separately as a component of income from continuing operations?

A $ 260,000

C $(255,000)
D $(350,000)

B $ 5,000

A

B $ 5,000

Event 1 - The gain on early retirement of bonds would be reported as a separate component of income from continuing operations. This is not unusual or infrequent as Fuqua frequently retires bonds early when the interest rates decline.

Event 2 - Hurricane losses have been frequent, and hence would be reported as a non-operating item. They will be treated as a loss from continuing operations.

Event 3 - The loss on disposal of a business segment is reported in discontinued operations, a separate component of net income after income from continuing operations.

The total amount of gain (loss) that should be reported by Fuqua Steel, separately as a component of income from continuing operations in the current year is hurricane loss of $255,000 and the gain on early retirement of bonds of $260,000 which gives a net gain of $5,000.

Option (A) is incorrect because it does not consider the loss from hurricane damage.
Option (C) is incorrect because it does not consider the gain from retiring bonds.
Option (D) is incorrect because loss from divestiture is not reported as part of income from continuing operations but reported separately in discontinued operations.

Question # 97 | Blueprint Area: 1 A iii : Income Statement/Statement of Profit or Loss and Statement of Comprehensive IncomeRelated Chapter: FAR-2 : Income Statement and Statement of Comprehensive Income

102
Q

On December 15, a U.S. company bought inventory from a European supplier. Payment is required in euros in 30 days. What exchange rate should be used to value the payable for this transaction at year-end?

A Exchange rate at settlement date.
B Exchange rate at purchase date.

D Weighted-average exchange rate for the year

C Exchange rate at year-end.

A

C Exchange rate at year-end.
An organization which purchases or sells goods in currencies other than in its functional currency is required to recognize foreign currency gain/loss on them at the settlement date using the spot rate. Where the settlement date is after the year-end, the foreign currency gain/loss is recognized at such year-end using the exchange rate at year-end.

Question # 174 | Blueprint Area: 1 A iii : Income Statement/Statement of Profit or Loss and Statement of Comprehensive IncomeRelated Chapter: FAR-11 : Foreign Currency Transactions

103
Q

A segment of Ace, Inc., was discontinued during the current year. In comparative financial statements for the previous year, Ace’s loss on disposal should

A Exclude comparative financial statements for contingent product warranty obligation costs.
B Include operating losses during the current fiscal year.
C Exclude additional pension costs associated with the decision to dispose.
D Include operating losses of the previous fiscal year up to the date a disposal plan was adopted.

A

D Include operating losses of the previous fiscal year up to the date a disposal plan was adopted.

In the period in which a component of an entity either has been disposed of or is classified as held for sale, the income statement of a business enterprise for current and prior periods shall report the results of operations of the component in discontinued operations in the period(s) in which they occur. Amounts reported in discontinued operations include the resolution of contingencies that arise from product warranty obligations and the settlement of employee benefit plan obligations (pension and other post-employment benefits), provided that the settlement is directly related to the disposal transaction.

Question # 109 | Blueprint Area: 1 A iii : Income Statement/Statement of Profit or Loss and Statement of Comprehensive IncomeRelated Chapter: FAR-2 : Income Statement and Statement of Comprehensive Income